Sie sind auf Seite 1von 68

Nat Res Cases

G.R. No. 164408

March 24, 2014

REPUBLIC OF THE PHILIPPINES, Petitioner,


vs.
ZURBARAN REALTY AND DEVELOPMENT CORPORATION, Respondent.
DECISION
BERSAMIN, J.:
An application for original registration of land of the public domain under Section 14(2) of Presidential
Decree (PD) No. 1529 must show not only that the land has previously been declared alienable and
disposable, but also that the land has been declared patrimonial property of the State at the onset of
the 30-year or 10-year period of possession and occupation required under the law on acquisitive
prescription. Once again, the Court applies this rule-as clarified in Heirs of Mario Malabanan v.
Republic1 in reviewing the decision promulgated on June 10, 2004, 2 whereby the Court of Appeals
(CA) granted the petitioner's application for registration of land.
Antecedents
On May 28, 1993, respondent Zurbaran Realty and Development Corporation filed in the Regional Trial
Court (RTC) in San Pedro, Laguna an application for original registration covering a 1,520 square meter
parcel of land situated in Barrio Banlic, Municipality of Cabuyao, Province of Laguna, denominated as
Lot 8017-A of Subdivision Plan CSD-04-006985-D, Cad. 455-D, Cabuyao Cadastre, 3 alleging that it had
purchased the land on March 9, 1992 from Jane de Castro Abalos, married to Jose Abalos,
for P300,000.00; that the land was declared for taxation purposes in the name of its predecessor-ininterest under Tax Declaration No. 22711; that there was no mortgage or encumbrance of any kind
affecting the land, nor was there any other person or entity having any interest thereon, legal or
equitable, adverse to that of the applicant; and that the applicant and its predecessors-in-interest had
been in open, continuous and exclusive possession and occupation of the land in the concept of an
owner.
Attached to the application were several documents, namely: (1) tracing cloth plan as approved by the
Land Management Division of the Department of Environment and Natural Resources (DENR); (2) blue
print copies of the tracing cloth plan; (3) copies of the technical description; (4) copies of Tax
Declaration No. 2711; and (5) copies of the Deed of Sale dated March 9, 1992.
The Republic, represented by the Director of Lands, opposed the application, arguing that the applicant
and its predecessors-in-interest had not been in open, continuous, exclusive and notorious possession
and occupation of the land since June 12, 1945; that the muniments of title and tax declaration
presented did not constitute competent and sufficient evidence of a bona fide acquisition of the land;
and that the land was a portion of the public domain, and, therefore, was not subject to private
appropriation.4
The RTC directed the Land Management Bureau, Manila; the Community Environment and Natural
Resources Office (CENRO) of Los Baos, Laguna; and the Land Management Sector and Forest
Management Bureau, Manila, to submit a status report on the land, particularly, on whether the land
was covered by a land patent, whether it was subject of a previously approved isolated survey, and
whether it was within a forest zone.5
In his memorandum to the DENR, Region IV (Lands Forestry Sector), and the Provincial Prosecutor of
Laguna, a copy of which was furnished the trial court, CENRO Officer Arnulfo Hernandez stated that the
land had been "verified to be within the Alienable and Disposable land under Land Classification

Project No. 23-A of Cabuyao, Laguna, certified and declared as such pursuant to the provisions of
Presidential Decree No. 705, as amended, under Forestry Administrative Order No. A-1627 dated
September 28, 1981 per BFD Map LC-3004." Attached to the memorandum was the inspection report
declaring that "the area is surrounded with concrete fence, three (3) buildings for employees
residence;" that the land was acquired through sale before the filing of the application; that the
applicant and its predecessors-in-interest had been in "continuous, open and peaceful occupation" of
the land, and that "no forestry interest is adversely affected." 6
CENRO Land Management Inspector/Investigator Rodolfo S. Gonzales reported that: (1) the land was
covered by a survey plan approved by the Regional Land Director/Land Registration Authority on May
25, 1988 pursuant to PD No. 239 dated July 9, 1975; (2) it consisted of 22,773 square meters and was
located in Barangay Banlic, Cabuyao, Laguna; (3) the area was entirely within the alienable and
disposable area; (4) it had never been forfeited in favor of the government for non-payment of taxes,
and had not been confiscated in connection with any civil or criminal cases; (5) it was not within a
previously patented property as certified to by the Register of Deeds, Calamba, Laguna; and (6) there
was no public land application filed for it by the applicant or any other persons as per verification from
the records unit of his office. The report further stated that a verification at the Office of the Municipal
Assessor showed that: (1) the land was declared for the first time in 1960 under Tax Declaration No.
6712 in the name of Enrique Hemedez with an area of 23,073 square meters; (2) it was now covered
by Tax Declaration No. 2253 issued in the name of the respondent; (3) the real property taxes had
been paid since 1968; and (4) it had not been earmarked for public or quasi-public purposes per
information from the District Engineer.
After inspection, it was also found that (1) the land was residential; (2) the respondent was in the
actual occupation and possession of the land; and (3) the land did not encroach upon an established
watershed, riverbank/bed protection, creek, right-of-way or park site or any area devoted to general
use or devoted to public service.7
A certification was issued by the Records Management Division of the Land Management Bureau
stating that it had no record of any kind of public land applications/land patents covering the parcel of
land subject of the application.8
The respondent presented Gloria P. Noel, its Vice President and Treasurer, who testified that the
respondent had purchased the land from Jane de Castro Abalos on March 9, 1992 for P300,000.00; that
the land had been declared for taxation purposes in the name of Abalos under Tax Declaration No.
22711; that after the sale, a new Tax Declaration had been issued in the name of the respondent, who
had meanwhile taken possession of the land by building a fence around it and introducing
improvements thereon; that the respondent had paid the real property taxes thereon since its
acquisition; that the respondents possession had been continuous, open and public; and that the land
was free from any lien or encumbrance; and that there was no adverse claimant to the land. 9
Engr. Edilberto Tamis attested that he was familiar with the land because it was a portion of Lot No.
8017 of Subdivision Plan Cad-455-D of the Cabuyao Cadastre, owned by Corazon Tapalla who had
acquired it from the Hemedez family; that Tapalla had sold a portion of Lot No. 8017 to Abalos and the
remaining portion to him; and that he had witnessed the sale of the land to the respondent. 10
The respondents final witness was Armando Espela who declared that he was a retired land overseer
residing in Barangay Banlic from birth; that he was familiar with the land which was part of a bigger
parcel of land owned by the Hemedez family; that his father, Toribio Espela, with his assistance, and
one Francisco Capacio worked on the land since 1960; that the entire landholding had originally been
sugarland, but was later on subdivided, sold, and resold until it ceased to be agricultural land; that, in
1982, the land was sold to Corazon Tapalla who hired him as the overseer; that as the overseer, he
fenced and cleared the area; that he was allowed to use the grassy portion for grazing purposes; that
in 1987, Tapalla sold part of the land to Abalos and the remaining portion to Engr. Tamis; that he

continued to oversee the land for the new owners; that Abalos then sold her portion to the respondent
in 1992; that since then, the respondent took possession of the land, and he then ceased to be the
overseer; that the possession by the Hemedez family and its successors-in-interest was open,
continuous, public and under claim of ownership; and that he did not know any person who claimed
ownership of the land other than those he and his father served as overseers. 11
Decision of the RTC
On May 12, 1997, the RTC rendered its decision, holding that the respondent and its predecessors-ininterest had been in open, public, peaceful, continuous, exclusive and adverse possession and
occupation of the land under a bona fide claim of ownership even prior to 1960 and, accordingly,
granted the application for registration, viz:
WHEREFORE, taking into consideration the evidence submitted by the applicant, this Court hereby
orders the confirmation and registration of title of the land described as Lot 8017-A of subdivision plan
Csd-04-006985-D, being a portion of Lot 8017 of subdivision plan Cad-455-D, Cabuyao Cadastre
situated at Barangay Banlic, Cabuyao, Laguna with an area of 1,520 square meters to be entered
under the name of the applicant Zurbaran Realty and Development Corporation, a corporation
organized and existing under the laws of the Philippines with office address at 33 M. Viola St., San
Francisco del Monte, Quezon City by the Land Registration Authority. After the decision shall become
final, let an order for the issuance of a decree of title be issued in favor of said applicant.
SO ORDERED.12
Judgment of the CA
The Republic appealed, arguing that the issue of whether the applicant and its predecessors-in-interest
had possessed the land within the required length of time could not be determined because there was
no evidence as to when the land had been declared alienable and disposable.
On June 10, 2004, the CA promulgated its judgment affirming the RTC, and concluded that the reports
made by the concerned government agencies and the testimonies of those familiar with the land in
question had buttressed the court a quos conclusion that the respondent and its predecessors-ininterest had been in open, public, peaceful, continuous, exclusive, and adverse possession and
occupation of the land under a bona fide claim of ownership even prior to 1960. 13
Issue
Hence, the Republic appeals the adverse judgment of the CA upon the following ground:
THE COURT OF APPEALS GRAVELY ERRED ON A QUESTION OF LAW WHEN IT AFFIRMED THE TRIAL
COURTS GRANT OF THE APPLICATION FOR ORIGINAL REGISTRATION DESPITE THE ABSENCE OF
EVIDENCE THAT RESPONDENT AND ITS PREDECESSORS-IN-INTEREST HAVE COMPLIED WITH THE
PERIOD OF POSSESSION AND OCCUPATION REQUIRED BY LAW. 14
The Republic contends that the respondent did not establish the time when the land covered by the
application for registration became alienable and disposable;15 that such detail was crucial because the
possession of the respondent and its predecessors-in-interest, for the purpose of determining whether
it acquired the property by prescription, should be reckoned from the time when the land was declared
alienable and disposable; and that prior to the declaration of the land of the public domain as alienable
and disposable, it was not susceptible to private ownership, and any possession or occupation at such
time could not be counted as part of the period of possession required under the law on prescription. 16
The respondent counters that whether it established when the property was declared alienable and
disposable and whether it complied with the 30-year required period of possession should not be

entertained anymore by the Court because: (a) these issues had not been raised in the trial court and
were being raised for the first time on appeal; and (b) factual findings of the trial court, especially
when affirmed by the CA, were binding and conclusive on this Court. At any rate, the respondent
insists that it had been in open, public, peaceful, continuous, and adverse possession of the property
for the prescribed period of 30 years as evidenced by the fact that the property had been declared for
taxation purposes in 1960 in the name of its predecessors-in-interest, and that such possession had
the effect of converting the land into private property and vesting ownership upon the respondent. 17
In reply, the Republic asserts that it duly opposed the respondents application for registration; that it
was only able to ascertain the errors committed by the trial court after the latter rendered its decision;
and that the burden of proof in land registration cases rested on the applicant who must prove its
ownership of the property being registered. The Republic maintains that the Court had the authority to
review and reverse the factual findings of the lower courts when the conclusion reached was not
supported by the evidence on record, as in this case.18
Ruling
The petition for review is meritorious.
Section 14 of P.D. No. 1529 enumerates those who may file an application for registration of land
based on possession and occupation of a land of the public domain, thus:
Section 14. Who may apply. The following persons may file in the proper Court of First Instance an
application for registration of title to land, whether personally or through their duly authorized
representatives:
(1) Those who by themselves or through their predecessors-in-interest have been in open, continuous,
exclusive and notorious possession and occupation of alienable and disposable lands of the public
domain under a bona fide claim of ownership since June 12, 1945, or earlier.
(2) Those who have acquired ownership of private lands by prescription under the provision of existing
laws.
xxxx
An application for registration under Section14(1) of P.D. No. 1529 must establish the following
requisites, namely: (a) the land is alienable and disposable property of the public domain; (b) the
applicant and its predecessors in interest have been in open, continuous, exclusive and notorious
possession and occupation of the land under a bona fide claim of ownership; and (c) the applicant and
its predecessors-in-interest have possessed and occupied the land since June 12, 1945, or earlier. The
Court has clarified in Malabanan19 that under Section14(1), it is not necessary that the land must have
been declared alienable and disposable as of June 12, 1945, or earlier, because the law simply requires
the property sought to be registered to be alienable and disposable at the time the application for
registration of title is filed. The Court has explained that a contrary interpretation would absurdly limit
the application of the provision "to the point of virtual inutility."
The foregoing interpretation highlights the distinction between a registration proceeding filed under
Section 14(1) of P.D. No. 1529 and one filed under Section 14(2) of P.D. No. 1529. According to
Malabanan:
Section 14(1) mandates registration on the basis of possession, while Section 14(2) entitles
registration on the basis of prescription. Registration under Section 14(1) is extended under the aegis
of the Property Registration Decree and the Public Land Act while registration under Section 14(2) is
made available both by the Property Registration Decree and the Civil Code. 20

In other words, registration under Section 14(1) of P.D. No. 1529 is based on possession and
occupation of the alienable and disposable land of the public domain since June 12, 1945 or earlier,
without regard to whether the land was susceptible to private ownership at that time. The applicant
needs only to show that the land had already been declared alienable and disposable at any time prior
to the filing of the application for registration.
On the other hand, an application under Section 14(2) of P.D. No. 1529 is based on acquisitive
prescription and must comply with the law on prescription as provided by the Civil Code. In that
regard, only the patrimonial property of the State may be acquired by prescription pursuant to the Civil
Code.21 For acquisitive prescription to set in, therefore, the land being possessed and occupied must
already be classified or declared as patrimonial property of the State. Otherwise, no length of
possession would vest any right in the possessor if the property has remained land of the public
dominion. Malabanan stresses that even if the land is later converted to patrimonial property of the
State, possession of it prior to such conversion will not be counted to meet the requisites of acquisitive
prescription.22 Thus, registration under Section 14(2) of P.D. No. 1529 requires that the land had
already been converted to patrimonial property of the State at the onset of the period of possession
required by the law on prescription.
An application for registration based on Section 14(2) of P.D. No. 1529 must, therefore, establish the
following requisites, to wit: (a) the land is an alienable and disposable, and patrimonial property of the
public domain; (b) the applicant and its predecessors-in-interest have been in possession of the land
for at least 10 years, in good faith and with just title, or for at least 30 years, regardless of good faith
or just title; and (c) the land had already been converted to or declared as patrimonial property of the
State at the beginning of the said 10-year or 30-year period of possession.
To properly appreciate the respondents case, we must ascertain under what provision its application
for registration was filed. If the application was filed under Section 14(1) of P.D. No. 1529, the
determination of the particular date when the property was declared alienable and disposable would
be unnecessary, inasmuch as proof showing that the land had already been classified as such at the
time the application was filed would be enough. If the application was filed under Section 14(2) of P.D.
No. 1529, the determination of the issue would not be crucial for, as earlier clarified, it was not the
declaration of the land as alienable and disposable that would make it susceptible to private ownership
by acquisitive prescription. Malabanan expounds thereon, thus Would such lands so declared
alienable and disposable be converted, under the Civil Code, from property of the public dominion into
patrimonial property? After all, by connotative definition, alienable and disposable lands may be the
object of the commerce of man; Article 1113 provides that all things within the commerce of man are
susceptible to prescription; and the same provision further provides that patrimonial property of the
State may be acquired by prescription.
Nonetheless, Article 422 of the Civil Code states that "[p]roperty of public dominion, when no longer
intended for public use or for public service, shall form part of the patrimonial property of the State." It
is this provision that controls how public dominion property may be converted into patrimonial
property susceptible to acquisition by prescription. After all, Article 420 (2) makes clear that those
property "which belong to the State, without being for public use, and are intended for some public
service or for the development of the national wealth" are public dominion property. For as long as the
property belongs to the State, although already classified as alienable or disposable, it remains
property of the public dominion if when it is "intended for some public service or for the development
of the national wealth."
Accordingly, there must be an express declaration by the State that the public dominion property is no
longer intended for public service or the development of the national wealth or that the property has
been converted into patrimonial. Without such express declaration, the property, even if classified as
alienable or disposable, remains property of the public dominion, pursuant to Article 420(2), and thus
incapable of acquisition by prescription. It is only when such alienable and disposable lands are

expressly declared by the State to be no longer intended for public service or for the development of
the national wealth that the period of acquisitive prescription can begin to run. Such declaration shall
be in the form of a law duly enacted by Congress or a Presidential Proclamation in cases where the
President is duly authorized by law.23
The respondents application does not enlighten as to whether it was filed under Section 14(1) or
Section 14(2) of P.D. No. 1529. The application alleged that the respondent and its predecessors-ininterest had been in open, continuous and exclusive possession and occupation of the property in the
concept of an owner, but did not state when possession and occupation commenced and the duration
of such possession. At any rate, the evidence presented by the respondent and its averments in the
other pleadings reveal that the application for registration was filed based on Section 14(2), not
Section 14(1) of P.D. No. 1529. The respondent did not make any allegation in its application that it had
been in possession of the property since June 12, 1945, or earlier, nor did it present any evidence to
establish such fact.1wphi1
With the application of the respondent having been filed under Section 14(2) of P.D. No. 1529, the
crucial query is whether the land subject of the application had already been converted to patrimonial
property of the State. In short, has the land been declared by law as no longer intended for public
service or the development of the national wealth?
The respondent may perhaps object to a determination of this issue by the Court for the same reason
that it objects to the determination of whether it established when the land was declared alienable and
disposable, that is, the issue was not raised in and resolved and by the trial court. But the objection
would be futile because the issue was actually raised in the trial court, as borne out by the Republic's
allegation in its opposition to the application to the effect "that the land is a portion of the public
domain not subject to prescription." In any case, the interest of justice dictates the consideration and
resolution of an issue that is relevant to another that was specifically raised. The rule that only theories
raised in the initial proceedings may be taken up by a party on appeal refers only to independent, not
concomitant, matters to support or oppose the cause of action. 24
Here, there is no evidence showing that the land in question was within an area expressly declared by
law either to be the patrimonial property of the State, or to be no longer intended for public service or
the development of the national wealth. The Court is left with no alternative but to deny the
respondent's application for registration.
WHEREFORE, the Court GRANTS the petition for review on certiorari; REVERSES and SETS ASIDE the
decision promulgated on June 10, 2004; and DISMISSES the respondent's application for original
registration of Lot 8017-A of Subdivision Plan CSD-04-006985-D, Cad. 455-D, of the Cabuyao Cadastre.
No pronouncement on costs of suit.
SO ORDERED.
LUCAS P. BERSAMIN
Associate Justice
G.R. No. 186639

February 5, 2014

REPUBLIC OF THE PHILIPPINES, Petitioner,


vs.
EMMANUEL C. CORTEZ, Respondent.
DECISION
REYES, J.:

Before this Court is a petition for review on certiorari1 under Rule 45 of the Rules of Court seeking to
annul and set aside the Decision2 dated February 17, 2009 of the Court of Appeals (CA) in CA-G.R. CV
No. 87505. The CA affirmed the Decision 3 dated February 7, 2006 of the Regional Trial Court (RTC) of
Pasig City, Branch 68, in LRC Case No. N-11496.
The Facts
On February 28, 2003, respondent Emmanuel C. Cortez (Cortez) filed with the RTC an application 4 for
judicial confirmation of title over a parcel of land located at Barangay (Poblacion) Aguho, P. Herrera
Street, Pateros, Metro Manila. The said parcel of land has an area of 110 square meters and more
particularly described as Lot No. 2697-B of the Pateros Cadastre. In support of his application, Cortez
submitted, inter alia, the following documents: (1) tax declarations for various years from 1966 until
2005; (2) survey plan of the property, with the annotation that the property is classified as alienable
and disposable; (3) technical description of the property, with a certification issued by a geodetic
engineer; (4) tax clearance certificate; (5) extrajudicial settlement of estate dated March 21, 1998,
conveying the subject property to Cortez; and (6) escritura de particion extrajudicial dated July 19,
1946, allocating the subject property to Felicisima Cotas Cortez mother.
As there was no opposition, the RTC issued an Order of General Default and Cortez was allowed to
present his evidence ex-parte.
Cortez claimed that the subject parcel of land is a portion of Lot No. 2697, which was declared for
taxation purposes in the name of his mother. He alleged that Lot No. 2697 was inherited by his mother
from her parents in 1946; that, on March 21, 1998, after his parents died, he and his siblings executed
an Extra-Judicial Settlement of Estate over the properties of their deceased parents and one of the
properties allocated to him was the subject property. He alleged that the subject property had been in
the possession of his family since time immemorial; that the subject parcel of land is not part of the
reservation of the Department of Environment and Natural Resources (DENR) and is, in fact, classified
as alienable and disposable by the Bureau of Forest Development (BFD).
Cortez likewise adduced in evidence the testimony of Ernesto Santos, who testified that he has known
the family of Cortez for over sixty (60) years and that Cortez and his predecessors-in-interest have
been in possession of the subject property since he came to know them.
On February 7, 2006, the RTC rendered a Decision, 5 which granted Cortez application for registration,
viz:
WHEREFORE, finding the application meritorious, the Court DECLARES, CONFIRMS, and ORDERS the
registration of the applicants title thereto.
As soon as this Decision shall have become final and after payment of the required fees, let the
corresponding Decrees be issued in the name of the applicant, Emmanuel C. Cortez.
Let copies of this Decision be furnished the Office of the Solicitor General, Land Registration Authority,
Land Management Bureau, and the Registry of Deeds of Rizal.
SO ORDERED.6
In granting Cortez application for registration of title to the subject property, the RTC made the
following ratiocinations:
From the foregoing, the Court finds that there is sufficient basis to grant the relief prayed for. It having
been established by competent evidence that the possession of the land being applied for by the
applicant and his predecessor-in-interest have been in open, actual, uninterrupted, and adverse

possession, under claim of title and in the concept of owners, all within the time prescribed by law, the
title of the applicant should be and must be AFFIRMED and CONFIRMED. 7
The Republic of the Philippines (petitioner), represented by the Office of the Solicitor General, appealed
to the CA, alleging that the RTC erred in granting the application for registration despite the failure of
Cortez to comply with the requirements for original registration of title. The petitioner pointed out that,
although Cortez declared that he and his predecessors-in-interest were in possession of the subject
parcel of land since time immemorial, no document was ever presented that would establish his
predecessors-in-interests possession of the same during the period required by law. That petitioner
claimed that Cortez assertion that he and his predecessors-in-interest had been in open, adverse, and
continuous possession of the subject property for more than thirty (30) years does not constitute wellneigh incontrovertible evidence required in land registration cases; that it is a mere claim, which
should not have been given weight by the RTC.
Further, the petitioner alleged that there was no certification from any government agency that the
subject property had already been declared alienable and disposable. As such, the petitioner claims,
Cortez possession of the subject property, no matter how long, cannot confer ownership or possessory
rights.
On February 17, 2009, the CA, by way of the assailed Decision, 8 dismissed the petitioners appeal and
affirmed the RTC Decision dated February 7, 2006. The CA ruled that Cortez was able to prove that the
subject property was indeed alienable and disposable, as evidenced by the declaration/notation from
the BFD.
Further, the CA found that Cortez and his predecessors-in-interest had been in open, continuous, and
exclusive possession of the subject property for more than 30 years, which, under Section 14(2) of
Presidential Decree (P.D.) No. 15299, sufficed to convert it to private property. Thus:
It has been settled that properties classified as alienable and disposable land may be converted into
private property by reason of open, continuous and exclusive possession of at least 30 years. Such
property now falls within the contemplation of "private lands" under Section 14(2) of PD 1529, over
which title by prescription can be acquired. Thus, under the second paragraph of Section 14 of PD
1529, those who are in possession of alienable and disposable land, and whose possession has been
characterized as open, continuous and exclusive for 30 years or more, may have the right to register
their title to such land despite the fact that their possession of the land commenced only after 12 June
1945. x x x
xxxx
While it is significant to note that applicant-appellees possession of the subject property can be traced
from his mothers possession of the same, the records, indeed, show that his possession of the subject
property, following Section 14(2) [of PD 1529], is to be reckoned from January 3, 1968, when the
subject property was declared alienable and disposable and not way back in 1946, the year when he
inherited the same from his mother. At any rate, at the time the application for registration was filed in
2003, there was already sufficient compliance with the requirement of possession. His possession of
the subject property has been characterized as open, continuous, exclusive and notorious possession
and occupation in the concept of an owner.10 (Citations omitted)
Hence, the instant petition.
The Issue
The sole issue to be resolved by the Court is whether the CA erred in affirming the RTC Decision dated
February 7, 2006, which granted the application for registration filed by Cortez.

The Courts Ruling


The petition is meritorious.
At the outset, the Court notes that the RTC did not cite any specific provision of law under which
authority Cortez application for registration of title to the subject property was granted. In granting
the application for registration, the RTC merely stated that "the possession of the land being applied
for by [Cortez] and his predecessor-in-interest have been in open, actual, uninterrupted, and adverse
possession, under claim of title and in the concept of owners, all within the time prescribed by
law[.]"11 On the other hand, the CA assumed that Cortez application for registration was based on
Section 14(2) of P.D. No. 1529. Nevertheless, Cortez, in the application for registration he filed with the
RTC, proffered that should the subject property not be registrable under Section 14(2) of P.D. No. 1529,
it could still be registered under Section 48(b) of Commonwealth Act No. 141 (C.A. No. 141), or the
Public Land Act, as amended by P.D. No. 1073 12 in relation to Section 14(1) of P.D. No. 1529. Thus, the
Court deems it proper to discuss Cortez application for registration of title to the subject property vis-vis the provisions of Section 14(1) and (2) of P.D. No. 1529.
Applicants for original registration of title to land must establish compliance with the provisions of
Section 14 of P.D. No. 1529, which pertinently provides that:
Sec. 14. Who may apply. The following persons may file in the proper Court of First Instance an
application for registration of title to land, whether personally or through their duly authorized
representatives:
(1) Those who by themselves or through their predecessors-in interest have been in open, continuous,
exclusive and notorious possession and occupation of alienable and disposable lands of the public
domain under a bona fide claim of ownership since June 12, 1945, or earlier.
(2) Those who have acquired ownership of private lands by prescription under the provision of existing
laws.
xxxx
After a careful scrutiny of the records of this case, the Court finds that Cortez failed to comply with the
legal requirements for the registration of the subject property under Section 14(1) and (2) of P.D. No.
1529.
Section 14(1) of P.D. No. 1529 refers to the judicial confirmation of imperfect or incomplete titles to
public land acquired under Section 48(b) of C.A. No. 141, as amended by P.D. No. 1073. "Under Section
14(1) [of P.D. No. 1529], applicants for registration of title must sufficiently establish first, that the
subject land forms part of the disposable and alienable lands of the public domain; second, that the
applicant and his predecessors-in-interest have been in open, continuous, exclusive, and notorious
possession and occupation of the same; and third, that it is under a bona fide claim of ownership since
June 12, 1945, or earlier."13
The first requirement was not satisfied in this case. To prove that the subject property forms part of the
alienable and disposable lands of the public domain, Cortez adduced in evidence a survey plan Csd-0000063314(conversion-subdivision plan of Lot 2697, MCadm 594-D, Pateros Cadastral Mapping) prepared
by Geodetic Engineer Oscar B. Fernandez and certified by the Lands Management Bureau of the DENR.
The said survey plan contained the following annotation:
This survey is inside L.C. Map No. 2623, Project No. 29, classified as alienable & disposable by the
Bureau of Forest Development on Jan. 3, 1968.

However, Cortez reliance on the foregoing annotation in the survey plan is amiss; it does not
constitute incontrovertible evidence to overcome the presumption that the subject property remains
part of the inalienable public domain. In Republic of the Philippines v. Tri-Plus Corporation, 15 the Court
clarified that, the applicant must at the very least submit a certification from the proper government
agency stating that the parcel of land subject of the application for registration is indeed alienable and
disposable, viz:
It must be stressed that incontrovertible evidence must be presented to establish that the land subject
of the application is alienable or disposable.
In the present case, the only evidence to prove the character of the subject lands as required by law is
the notation appearing in the Advance Plan stating in effect that the said properties are alienable and
disposable. However, this is hardly the kind of proof required by law. To prove that the land subject of
an application for registration is alienable, an applicant must establish the existence of a positive act
of the government such as a presidential proclamation or an executive order, an administrative action,
investigation reports of Bureau of Lands investigators, and a legislative act or statute. The applicant
may also secure a certification from the Government that the lands applied for are alienable and
disposable. In the case at bar, while the Advance Plan bearing the notation was certified by the Lands
Management Services of the DENR, the certification refers only to the technical correctness of the
survey plotted in the said plan and has nothing to do whatsoever with the nature and character of the
property surveyed. Respondents failed to submit a certification from the proper government agency to
prove that the lands subject for registration are indeed alienable and disposable. 16 (Citations omitted
and emphasis ours)
Similarly, in Republic v. Roche,17 the Court declared that:
Respecting the third requirement, the applicant bears the burden of proving the status of the land. In
this connection, the Court has held that he must present a certificate of land classification status
issued by the Community Environment and Natural Resources Office (CENRO) or the Provincial
Environment and Natural Resources Office (PENRO) of the DENR. He must also prove that the DENR
Secretary had approved the land classification and released the land as alienable and disposable, and
that it is within the approved area per verification through survey by the CENRO or PENRO. Further, the
applicant must present a copy of the original classification approved by the DENR Secretary and
certified as true copy by the legal custodian of the official records. These facts must be established by
the applicant to prove that the land is alienable and disposable.
Here, Roche did not present evidence that the land she applied for has been classified as alienable or
disposable land of the public domain. She submitted only the survey map and technical description of
the land which bears no information regarding the lands classification. She did not bother to establish
the status of the land by any certification from the appropriate government agency. Thus, it cannot be
said that she complied with all requisites for registration of title under Section 14(1) of P.D.
1529.18 (Citations omitted and emphasis ours)
The annotation in the survey plan presented by Cortez is not the kind of evidence required by law as
proof that the subject property forms part of the alienable and disposable land of the public domain.
Cortez failed to present a certification from the proper government agency as to the classification of
the subject property. Cortez likewise failed to present any evidence showing that the DENR Secretary
had indeed classified the subject property as alienable and disposable. Having failed to present any
incontrovertible evidence, Cortez claim that the subject property forms part of the alienable and
disposable lands of the public domain must fail.
Anent the second and third requirements, the Court finds that Cortez likewise failed to establish the
same.1wphi1 Cortez failed to present any evidence to prove that he and his predecessors-in-interest
have been in open, continuous, exclusive, and notorious possession and occupation of the subject
property since June 12, 1945, or earlier. Cortez was only able to present oral and documentary

evidence of his and his mothers ownership and possession of the subject property since 1946, the
year in which his mother supposedly inherited the same.
Other than his bare claim that his family possessed the subject property since time immemorial, Cortez
failed to present any evidence to show that he and his predecessors-in-interest indeed possessed the
subject property prior to 1946; it is a mere claim and not factual proof of possession. "It is a rule that
general statements that are mere conclusions of law and not factual proof of possession are unavailing
and cannot suffice. An applicant in a land registration case cannot just harp on mere conclusions of law
to embellish the application but must impress thereto the facts and circumstances evidencing the
alleged ownership and possession of the land." 19
Further, the earliest tax declaration presented by Cortez was only in 1966. Cortez failed to explain why,
despite his claim that he and his predecessors-in-interest have been in possession of the subject
property since time immemorial, it was only in 1966 that his predecessors-in-interest started to declare
the same for purposes of taxation.
That Cortez and his predecessors-in-interest have been in possession of the subject property for fiftyseven (57) years at the time he filed his application for registration in 2003 would likewise not entitle
him to registration thereof under Section 14(2) of P.D. No. 1529.
Section 14(2) of P.D. No. 1529 sanctions the original registration of lands acquired by prescription
under the provisions of existing laws. "As Section 14(2) [of P.D. No. 1529] categorically provides, only
private properties may be acquired thru prescription and under Articles 420 and 421 of the Civil Code,
only those properties, which are not for public use, public service or intended for the development of
national wealth, are considered private."20
In Heirs of Mario Malabanan v. Republic,21 the Court however clarified that lands of the public domain
that are patrimonial in character are susceptible to acquisitive prescription and, accordingly, eligible
for registration under Section 14(2) of P.D. No. 1529, viz:
The Civil Code makes it clear that patrimonial property of the State may be acquired by private
persons through prescription. This is brought about by Article 1113, which states that "[a]ll things
which are within the commerce of man are susceptible to prescription," and that property of the State
or any of its subdivisions not patrimonial in character shall not be the object of prescription."
There are two modes of prescription through which immovables may be acquired under the Civil
Code.1wphi1 The first is ordinary acquisitive prescription, which, under Article 1117, requires
possession in good faith and with just title; and, under Article 1134, is completed through possession
of ten (10) years. There is nothing in the Civil Code that bars a person from acquiring patrimonial
property of the State through ordinary acquisitive prescription, nor is there any apparent reason to
impose such a rule. At the same time, there are indispensable requisitesgood faith and just title. The
ascertainment of good faith involves the application of Articles 526, 527, and 528, as well as Article
1127 of the Civil Code, provisions that more or less speak for themselves. 22 (Citation omitted and
emphasis ours)
The Court nevertheless emphasized that there must be an official declaration by the State that the
public dominion property is no longer intended for public use, public service, or for the development of
national wealth before it can be acquired by prescription; that a mere declaration by government
officials that a land of the public domain is already alienable and disposable would not suffice for
purposes of registration under Section 14(2) of P.D. No. 1529. The Court further stressed that the
period of acquisitive prescription would only begin to run from the time that the State officially
declares that the public dominion property is no longer intended for public use, public service, or for
the development of national wealth. Thus:

Let us now explore the effects under the Civil Code of a declaration by the President or any duly
authorized government officer of alienability and disposability of lands of the public domain. Would
such lands so declared alienable and disposable be converted, under the Civil Code, from property of
the public dominion into patrimonial property? After all, by connotative definition, alienable and
disposable lands may be the object of the commerce of man; Article 1113 provides that all things
within the commerce of man are susceptible to prescription; and the same provision further provides
that patrimonial property of the State may be acquired by prescription.
Nonetheless, Article 422 of the Civil Code states that "[p]roperty of public dominion, when no longer
intended for public use or for public service, shall form part of the patrimonial property of the State." It
is this provision that controls how public dominion property may be converted into patrimonial
property susceptible to acquisition by prescription. After all, Article 420 (2) makes clear that those
property "which belong to the State, without being for public use, and are intended for some public
service or for the development of the national wealth" are public dominion property. For as long as the
property belongs to the State, although already classified as alienable or disposable, it remains
property of the public dominion if when it is "intended for some public service or for the development
of the national wealth."
Accordingly, there must be an express declaration by the State that the public dominion property is no
longer intended for public service or the development of the national wealth or that the property has
been converted into patrimonial. Without such express declaration, the property, even if classified as
alienable or disposable, remains property of the public dominion, pursuant to Article 420(2), and thus
incapable of acquisition by prescription. It is only when such alienable and disposable lands are
expressly declared by the State to be no longer intended for public service or for the development of
the national wealth that the period of acquisitive prescription can begin to run. Such declaration shall
be in the form of a law duly enacted by Congress or a Presidential Proclamation in cases where the
President is duly authorized by law.23 (Emphasis supplied)
In Republic v. Rizalvo,24 the Court deemed it appropriate to reiterate the ruling in Malabanan, viz:
On this basis, respondent would have been eligible for application for registration because his claim of
ownership and possession over the subject property even exceeds thirty (30) years. However, it is
jurisprudentially clear that the thirty (30)-year period of prescription for purposes of acquiring
ownership and registration of public land under Section 14 (2) of P.D. No. 1529 only begins from the
moment the State expressly declares that the public dominion property is no longer intended for public
service or the development of the national wealth or that the property has been converted into
patrimonial. x x x.25 (Citation omitted and emphasis ours)
Accordingly, although lands of the public domain that are considered patrimonial may be acquired by
prescription under Section 14(2) of P.D. No. 1529, before acquisitive prescription could commence, the
property sought to be registered must not only be classified as alienable and disposable; it must also
be declared by the State that it is no longer intended for public use, public service or the development
of the national wealth. Thus, absent an express declaration by the State, the land remains to be
property of public dominion.26
The Court finds no evidence of any official declaration from the state attesting to the patrimonial
character of the subject property. Cortez failed to prove that acquisitive prescription has begun to run
against the State, much less that he has acquired title to the subject property by virtue thereof. It is of
no moment that Cortez and his predecessors-in-interest have been in possession of the subject
property for 57 years at the time he applied for the registration of title thereto. "[l]t is not the
notorious, exclusive and uninterrupted possession and occupation of an alienable and disposable
public land for the mandated periods that converts it to patrimonial. The indispensability of an official
declaration that the property is now held by the State in its private capacity or placed within the
commerce of man for prescription to have any effect against the State cannot be overemphasized. " 27

WHEREFORE, in consideration of the foregoing disquisitions, the instant petition is GRANTED. The
Decision dated February 17, 2009 of the Court of Appeals in CA-G.R. CV No. 87505, which affirmed the
Decision dated February 7, 2006 of the Regional Trial Court of Pasig City, Branch 68, in LRC Case No. N11496, is hereby REVERSED and SET ASIDE. The Application for Registration of Emmanuel C. Cortez in
LRC Case No. N-11496 is DENIED for lack of merit.
SO ORDERED.
BIENVENIDO L. REYES
Associate Justice
G.R. No. 191590

April 21, 2014

REPUBLIC OF THE PHILIPPINES, Petitioner,


vs.
TRANSUNION CORPORATION, Respondent.
DECISION
PERLAS-BERNABE, J.:
Assailed in this petition for review on certiorari 1 are the Decision2 dated October 9, 2009 and the
Resolution3dated March 10, 2010 of the Court of Appeals (CA) in CA-G.R. SP No,. 106544 which set
aside the Order4 dated August 14, 2008 of the Regional Trial Court of Imus, Cavite, Branch 22 (RTC)
denying the motion to dismiss filed by respondent Transunion Corporation (Transunion) in Civil Case
No. 2085-08.
The Facts
On April 30, 1999, Leticia Salamat (Salamat) filed an Application to Purchase Friar Lands, 5 specifically
Lot No. 5741 of the Imus Estate (Lot No. 5741), with the Community Environment and Natural
Resources Office (CENRO) of the Department of Environment and Natural Resources (DENR). 6 Her
application was subsequently indorsed to the Land Management Bureau (LMB) for final
action.7 Thereafter, Salamat was informed that Lot No. 5741 was already covered by Transfer
Certificate of Title (TCT) No. T-6167408 in the name of Transunion.9 This prompted Salamat to file, on
June 27, 2000, a Protest10 against Transunion with the LMB , docketed as LMB Case No. 114, alleging
that TCT No. T-616740 was obtained through fraud considering that no deed of conveyance was issued
by the LMB for Lot No. 5741 in the name of any person. 11 In this relation, Salamat averred that she and
her family had been in continuous possession and occupation of the said lot since time immemorial
and had even introduced improvements thereon. She likewise stated that it was only after the LMB
favorably endorsed her application, that it was discovered that Lot No. 5741 was already covered by
TCT No. T-616740.12
On September 13, 2000, LMB OIC-Director Ernesto D. Adobo, Jr. (Director Adobo) issued Special Order
No. 2000-175, designating Atty. Rogelio C. Mandar (Atty. Mandar) and one Carlito Manga, Jr. to conduct
a formal investigation in order to determine the veracity of the allegations contained in Salamats
protest pursuant to Lands Office Circular No. 68 (LC 68).13
On November 8, 2000, Transunion filed with the LMB a motion to dismiss, alleging that Salamat had no
legal personality to attack the validity of Transunion's title, and that it is the RTC which has jurisdiction
to try and decide cases involving cancellation of titles. 14 On February 8, 2001, Director Adobo denied
the motion to dismiss and directed Atty. Mandar to proceed with the investigation. 15
After due proceedings, Atty. Mandar issued an investigation report 16 dated July 8, 2003 (investigation
report) addressed to "The Director Thru the OIC-Chief Legal Division, Lands Management

Bureau,"17 recommending that steps be taken before a competent court of justice for the annulment of
TCT No. T-616740 and the reversion of Lot No. 5741 to the government. 18 The recommendation was
adopted by the Legal Division in its memorandum19dated November 2003 addressed to the Director,
which was later approved by LMB Director Concordio D. Zuiga (LMB Director). 20
Neither Salamat nor Transunion were furnished copies of the investigation report or memorandum. 21
On April 20, 2004, the DENR transmitted to the Office of the Solicitor General (OSG) the entire records
of LMB Case No. 114.22 Accordingly, a complaint for cancellation of title and/or reversion, docketed as
Civil Case No. 2085-08 (reversion complaint), was filed by herein petitioner the Republic of the
Philippines (Republic) against Transunion and its predecessors-in-interest, with the RTC. 23
In response, Transunion filed a motion to dismiss24 on the ground that the filing of the reversion
complaint was premature. Specifically, it argued that a condition precedent for the filing of the
complaint had not been complied with that is, the failure of the LMB to notify Transunion of its
recommendation in the investigation report thereby depriving it the opportunity to seek a
reconsideration or an appeal of the same, and ultimately resulting in a failure to exhaust
administrative remedies. Hinged on the foregoing theory, Transunion further claimed that the reversion
complaint stated no cause of action.
The RTC Ruling
In an Order25 dated August 14, 2008, the RTC denied Transunion's motion to dismiss.
It held that the investigation report was merely a recommendation for a "possible action that should be
taken" by the LMB Director.26 Accordingly, Atty. Mandars actions were not in the exercise of a quasijudicial function, hence, not subject to a motion for reconsideration or appeal. It is in this regard that
the RTC concluded there was any failure to comply with a condition precedent. 27
Relatedly, the RTC ruled that the Republics reversion complaint did state a cause of action based on
its examination of the allegations and arguments stated therein. 28
Dissatisfied, Transunion elevated the matter on certiorari. 29
The CA Ruling
In a Decision30 dated October 9, 2009, the CA reversed the RTC's ruling, observing that no decision was
rendered in LMB Case No. 114 and that Transunion was denied the right to be informed of the DENR's
official action as well as the opportunity to contest said action. As such, it pronounced that the filing of
the Republics reversion complaint was premature and that the latters failure to exhaust
administrative remedies was fatal to its cause of action.31
At odds with the CA's Decision, the Republic filed a motion for reconsideration, 32 which was, however,
denied by the CA in a Resolution33 dated March 10, 2010, hence, this petition.
The Issue Before the Court
The sole issue for the Court's resolution is whether or not the CA correctly granted Transunions
petition for certiorari against the RTC's order denying the latters motion to dismiss.
The Court's Ruling
The petition is meritorious.
An order denying a motion to dismiss is an interlocutory order which neither terminates nor finally
disposes of a case as it leaves something to be done by the court before the case is finally decided on

the merits. Thus, as a general rule, the denial of a motion to dismiss cannot be questioned in a special
civil action for certiorari which is a remedy designed to correct errors of jurisdiction and not errors of
judgment. However, when the denial of the motion to dismiss is tainted with grave abuse of discretion,
the grant of the extraordinary remedy of certiorari may be justified. By grave abuse of discretion is
meant such capricious and whimsical exercise of judgment that is equivalent to lack of jurisdiction. The
abuse of discretion must be grave as where the power is exercised in an arbitrary or despotic manner
by reason of passion or personal hostility, and must be so patent and gross as to amount to an evasion
of positive duty or to a virtual refusal to perform the duty enjoined by or to act at all in contemplation
of law.34
In the present case, the Court finds that the RTC did not commit any grave abuse of discretion in
denying Transunions motion to dismiss considering that the latters further reconsideration or appeal
of the investigation report was not a condition precedent to the filing of the Republics reversion
complaint. As such, there was no violation of the rule on exhaustion of administrative remedies nor can
it be said that the reversion complaint stated no cause of action.
To elaborate, the rule on exhaustion of administrative remedies provides that if a remedy within the
administrative machinery can still be resorted to by giving the administrative officer concerned every
opportunity to decide on a matter that comes within his jurisdiction, then such remedy should be
exhausted first before the courts judicial power can be sought. The underlying principle of the rule
rests on the presumption that the administrative agency, if afforded a complete chance to pass upon
the matter will decide the same correctly.35
Transunion reiterates that the Republics reversion complaint should be dismissed on the ground that it
was not notified of the investigation report, recommending that steps be taken before a competent
court of justice for the annulment of TCT No. T-616740 and the reversion of Lot No. 5741 to the
government. It argues that it should have been notified of said report and recommendation so that it
would have been able to contest the same on reconsideration or on appeal. Without having been able
to avail of these remedies, Transunion decries a violation of the rule on exhaustion of administrative
remedies and, perforce, prays that the Republics reversion complaint be dismissed.
Transunion is mistaken.
As may be gleaned from the records,36 the LMB proceeding subject of Transunions motion to dismiss
was merely investigative in nature since it was conducted as a fact-finding/recommendatory
procedure, meant only to determine whether or not the LMB Director should initiate reversion
proceedings. This proceeding was taken under LC 68, captioned as "Investigation of Claims and
Conflicts."37 Section 15 of LC 68, which states the parameters to be observed regarding the report and
recommendation resulting from the said investigation, is bereft of any indication that the remedies of
reconsideration or a further appeal is available to a party disagreeing with the same, viz.:
SEC. 15. Report of Investigation. Within 30 days from the date of termination of the investigation, the
hearing officer concerned shall render his report on the case to the Regional Executive Director. He
shall forward together with his report the complete records of the proceedings, evidence of the parties
and such other papers, documents and record relevant thereto.
The report of the investigation should contain the following:
1. Caption and title of the case;
2. Statement as to how the case arose and by virtue of whose authority investigation was conducted;
3. Statement that notices have been sent to parties and how they were notified;
4. Statement as to when and where formal investigation was conducted;

5. Parties appearing thereat including the counsel representing them, if any, and their addresses;
6. Findings in the ocular inspection including the description of improvements and sketch of the land
showing the portion contested and statement that efforts had been exerted to settle the case amicably
between the parties;
7. Summary of the testimony of the parties and witnesses and enumeration and substance of the
documentary evidence submitted by them;
8. Observation on the case including the demeanor of the persons who testified thereat;
9. Recommendations.
The report must be prepared immediately after the hearing while the matter is still fresh in the
investigators mind. In no case shall such report be a brief in support of one of the parties or contain a
discussion of the law applicable to the case. The investigator shall present only the facts as he
gathered them at the investigation.38 (Emphases supplied)
Transunion confuses the investigation report and the recommendation made therein with an action of
the LMB Regional Executive Director found in Section 3.1 of the Manual on Settlement of Land
Disputes39 (Land Disputes Manual) characterized as follows:
3.1 Matters covered by decisions or orders of the Regional Executive Director.
All actions of the Regional Executive Director in approving, rejecting; reinstating or cancelling a public
land application, or deciding a conflict, dismissing a claim or determining any matters in relation
thereto, shall be published in the form of a judicial decision or order. All parties concerned or their
attorneys or representatives and the Central Office, Lands Management Bureau shall be furnished
copies of the decision or order.40 (Emphases and underscoring supplied)
The distinctions between an investigative function such as that taken by the LMB in this case and
an adjudicative function such as that described in Section 3.1 above have been extensively
discussed by the Court in the case of Cario v. Commission on Human Rights, 41 to wit:
"Investigate," commonly understood, means to examine, explore, inquire or delve or probe into,
research on, study. The dictionary definition of "investigate" is "to observe or study closely; inquire into
systematically: to search or inquire into x x x to subject to an official probe x x x: to conduct an
official inquiry." The purpose of [an] investigation, of course is to discover, to find out, to learn, obtain
information. Nowhere included or intimated is the notion of settling, deciding or resolving a
controversy involved in the facts inquired into by application of the law to the facts established by the
inquiry.
The legal meaning of "investigate" is essentially the same: "(t)o follow up step by step by patient
inquiry or observation. To trace or track; to search into; to examine and inquire into with care and
accuracy; to find out by careful inquisition; examination; the taking of evidence; a legal inquiry;" "to
inquire; to make an investigation," "investigation" being in turn described as "(a)n administrative
function, the exercise of which ordinarily does not require a hearing. 2 Am J2d Adm L Sec. 257; x x x an
inquiry, judicial or otherwise, for the discovery and collection of facts concerning a certain matter or
matters."
"Adjudicate," commonly or popularly understood, means to adjudge, arbitrate, judge, decide,
determine, resolve, rule on, settle. The dictionary defines the term as "to settle finally (the rights and
duties of parties to a court case) on the merits of issues raised: x x x to pass judgment on:
settle judicially: x x x act as judge." And "adjudge" means "to decide or rule upon as a judge or with
judicial or quasi-judicial powers: x x x to award or grant judicially in a case of controversy x x x."

In the legal sense, "adjudicate" means: "To settle in the exercise of judicial authority. To determine
finally. Synonymous with adjudge in its strictest sense;" and "adjudge" means: "To pass on judicially, to
decide, settle, or decree, or to sentence or condemn. x x x Implies a judicial determination of a fact,
and the entry of a judgment."42(Emphases supplied)
Based on Section 3.1 of the Land Disputes Manual as above-cited, it is clear that it is the action of the
Regional Executive Director in "approving, rejecting; reinstating or cancelling a public land application,
or deciding a conflict, dismissing a claim or determining any matters in relation thereto" which is
required to be "published in the form of a judicial decision or order," and from which the remedies of
reconsideration and appeal may be taken pursuant to Section 3.2 of the Land Disputes Manual, viz.:
3.2 Appeal from decision or order of the Regional Executive Director
An appeal from a decision or order of the Regional Executive Director to the Secretary of Environment
and Natural Resources shall be within a period of thirty (30) days to be counted from the date the
interested party received the notice thereof unless a motion for reconsideration is filed within the said
period, in which case, appeal shall be made within thirty (30) days from his receipt of notice of the
order or decision of the Regional Executive Director disposing of the motion [for] reconsideration. The
notice of appeal may be delivered or sent to (1) the Community Environment and Natural Resources
Officer or provincial Environment and Natural Resources Officer, (2) the Regional Director deciding the
case, (3) the Secretary or Undersecretary of Environment and Natural Resources. On receipt thereof,
the Community Environment and Natural Resources Officer or Provincial Environment and Natural
Resources Officer concerned shall note thereon the date when it is received by him and shall forward
the same without delay to the Secretary of Environment and Natural Resources. In case the appeal is
delivered or sent to the Director of Lands, he shall forward the same together with the official records
pertinent to the case to the Secretary of Environment and Natural Resources. In case the appeal is sent
directly to the Secretary of Environment and Natural Resources, the Regional Executive Director shall
be served with a notice of appeal. The time of filing the appeal with any one of the said officers, as
specified in Section 16 thereof, shall be considered as the time when the appeal is taken. 43 (Emphases
and underscoring supplied)
Said course of action is that which may be considered as a form of "adjudication," resulting as it would
in the settlement of a public land application, or a decision on a public land conflict or claim. Given its
nature, the Land Disputes Manual then requires that it be published in the form of a judicial decision or
order and, concomitantly, be subject to further reconsideration and/or appeal. This action is clearly
different from the LMB proceeding subject of this case which, as earlier stated, is merely investigative
in nature. As further explained by the Republic in its petition, "[t]he investigation carried out by the
Director of Lands merely determines the propriety of initiating reversion proceedings and is an internal
procedure within the exclusive discretion of the LMB." 44 With this in mind, the latter proceeding and the
recommendation reached thereby cannot then be considered to be governed by Sections 3.1 and 3.2
of the Land Disputes Manual which respectively provide the requirement of notice and the remedies of
reconsideration or appeal. Corollarily, since these administrative remedies were not available to
Transunion against the investigation report and recommendation, there was thus no violation of the
rule on exhaustion of administrative remedies. As such, Transunions claim that a condition precedent
was left unfulfilled was properly debunked by the RTC.
Further, barring any violation of the doctrine of exhaustion of administrative remedies as abovediscussed, Transunions assertion on the reversion complaints purported failure to state a cause of
action (or properly speaking, the Republics lack of cause of action) 45 hinged as it is solely on the
same theory - was also properly denied.1wphi1
Finally, the Court finds that there was no violation of Transunion's right to administrative due process
since, as the Republic pointed out, not only did it file an answer, but it also presented its evidence and
formally offered the .same.46 It is well-established that the touchstone of due process is the opportunity

to be heard.47 This Transunion was unquestionably afforded in this case, despite having been denied
the remedies of reconsideration and appeal which, however, remain unavailable, either by statute or
regulation, against the investigation report and recomme'ndation assailed herein. At any rate, lack of
administrative due process, on the assumption of its truth, is not a ground for a motion to
dismiss;48 hence, the RTC's ruling was altogether proper.
For the reasons above-stated, the Court therefore concludes that the RTC did not gravely abuse its
discretion in denying Transunion 's motion to dismiss against the Republic's reversion complaint. As
such, the CA committed a reversible error in granting Transunion's petition for certiorari, warranting
the reversal of its Decision.
WHEREFORE the petition is GRANTED. The Decision dated October 9, 2009 and the Resolution dated
March 10, 2010 of the Court of Appeals in .CA-G.R. SP No. 106544 are hereby REVERSED and SET
ASIDE.
SO ORDERED.
ESTELA M. PERLAS-BERNABE
Associate Justice
[G.R. No. 135385. December 6, 2000]
ISAGANI CRUZ and CESAR EUROPA, petitioners, vs. SECRETARY OF ENVIRONMENT AND
NATURAL RESOURCES, SECRETARY OF BUDGET AND MANAGEMENT and CHAIRMAN and
COMMISSIONERS OF THE NATIONAL COMMISSION ON INDIGENOUS PEOPLES, respondents.
PER CURIAM:
Petitioners Isagani Cruz and Cesar Europa brought this suit for prohibition and mandamus as citizens
and taxpayers, assailing the constitutionality of certain provisions of Republic Act No. 8371 (R.A. 8371),
otherwise known as the Indigenous Peoples Rights Act of 1997 (IPRA), and its Implementing Rules and
Regulations (Implementing Rules).
In its resolution of September 29, 1998, the Court required respondents to comment. [1] In compliance,
respondents Chairperson and Commissioners of the National Commission on Indigenous Peoples
(NCIP), the government agency created under the IPRA to implement its provisions, filed on October
13, 1998 their Comment to the Petition, in which they defend the constitutionality of the IPRA and pray
that the petition be dismissed for lack of merit.
On October 19, 1998, respondents Secretary of the Department of Environment and Natural Resources
(DENR) and Secretary of the Department of Budget and Management (DBM) filed through the Solicitor
General a consolidated Comment. The Solicitor General is of the view that the IPRA is partly
unconstitutional on the ground that it grants ownership over natural resources to indigenous peoples
and prays that the petition be granted in part.
On November 10, 1998, a group of intervenors, composed of Sen. Juan Flavier, one of the authors of
the IPRA, Mr. Ponciano Bennagen, a member of the 1986 Constitutional Commission, and the leaders
and members of 112 groups of indigenous peoples (Flavier, et. al), filed their Motion for Leave to
Intervene. They join the NCIP in defending the constitutionality of IPRA and praying for the dismissal of
the petition.
On March 22, 1999, the Commission on Human Rights (CHR) likewise filed a Motion to Intervene and/or
to Appear as Amicus Curiae. The CHR asserts that IPRA is an expression of the principle of parens
patriae and that the State has the responsibility to protect and guarantee the rights of those who are

at a serious disadvantage like indigenous peoples. For this reason it prays that the petition be
dismissed.
On March 23, 1999, another group, composed of the Ikalahan Indigenous People and the Haribon
Foundation for the Conservation of Natural Resources, Inc. (Haribon, et al.), filed a motion to Intervene
with attached Comment-in-Intervention. They agree with the NCIP and Flavier, et al. that IPRA is
consistent with the Constitution and pray that the petition for prohibition and mandamus be dismissed.
The motions for intervention of the aforesaid groups and organizations were granted.
Oral arguments were heard on April 13, 1999. Thereafter, the parties and intervenors filed their
respective memoranda in which they reiterate the arguments adduced in their earlier pleadings and
during the hearing.
Petitioners assail the constitutionality of the following provisions of the IPRA and its Implementing
Rules on the ground that they amount to an unlawful deprivation of the States ownership over lands of
the public domain as well as minerals and other natural resources therein, in violation of the regalian
doctrine embodied in Section 2, Article XII of the Constitution:
(1) Section 3(a) which defines the extent and coverage of ancestral domains, and Section 3(b) which,
in turn, defines ancestral lands;
(2) Section 5, in relation to section 3(a), which provides that ancestral domains including inalienable
public lands, bodies of water, mineral and other resources found within ancestral domains are private
but community property of the indigenous peoples;
(3) Section 6 in relation to section 3(a) and 3(b) which defines the composition of ancestral domains
and ancestral lands;
(4) Section 7 which recognizes and enumerates the rights of the indigenous peoples over the ancestral
domains;
(5) Section 8 which recognizes and enumerates the rights of the indigenous peoples over the ancestral
lands;
(6) Section 57 which provides for priority rights of the indigenous peoples in the harvesting, extraction,
development or exploration of minerals and other natural resources within the areas claimed to be
their ancestral domains, and the right to enter into agreements with nonindigenous peoples for the
development and utilization of natural resources therein for a period not exceeding 25 years,
renewable for not more than 25 years; and
(7) Section 58 which gives the indigenous peoples the responsibility to maintain, develop, protect and
conserve the ancestral domains and portions thereof which are found to be necessary for critical
watersheds, mangroves, wildlife sanctuaries, wilderness, protected areas, forest cover or reforestation.
[2]

Petitioners also content that, by providing for an all-encompassing definition of ancestral domains and
ancestral lands which might even include private lands found within said areas, Sections 3(a) and 3(b)
violate the rights of private landowners.[3]
In addition, petitioners question the provisions of the IPRA defining the powers and jurisdiction of the
NCIP and making customary law applicable to the settlement of disputes involving ancestral domains
and ancestral lands on the ground that these provisions violate the due process clause of the
Constitution.[4]
These provisions are:

(1) sections 51 to 53 and 59 which detail the process of delineation and recognition of ancestral
domains and which vest on the NCIP the sole authority to delineate ancestral domains and ancestral
lands;
(2) Section 52[i] which provides that upon certification by the NCIP that a particular area is an
ancestral domain and upon notification to the following officials, namely, the Secretary of Environment
and Natural Resources, Secretary of Interior and Local Governments, Secretary of Justice and
Commissioner of the National Development Corporation, the jurisdiction of said officials over said area
terminates;
(3) Section 63 which provides the customary law, traditions and practices of indigenous peoples shall
be applied first with respect to property rights, claims of ownership, hereditary succession and
settlement of land disputes, and that any doubt or ambiguity in the interpretation thereof shall be
resolved in favor of the indigenous peoples;
(4) Section 65 which states that customary laws and practices shall be used to resolve disputes
involving indigenous peoples; and
(5) Section 66 which vests on the NCIP the jurisdiction over all claims and disputes involving rights of
the indigenous peoples.[5]
Finally, petitioners assail the validity of Rule VII, Part II, Section 1 of the NCIP Administrative Order No.
1, series of 1998, which provides that the administrative relationship of the NCIP to the Office of the
President is characterized as a lateral but autonomous relationship for purposes of policy and program
coordination. They contend that said Rule infringes upon the Presidents power of control over
executive departments under Section 17, Article VII of the Constitution. [6]
Petitioners pray for the following:
(1) A declaration that Sections 3, 5, 6, 7, 8, 52[I], 57, 58, 59, 63, 65 and 66 and other related
provisions of R.A. 8371 are unconstitutional and invalid;
(2) The issuance of a writ of prohibition directing the Chairperson and Commissioners of the NCIP to
cease and desist from implementing the assailed provisions of R.A. 8371 and its Implementing Rules;
(3) The issuance of a writ of prohibition directing the Secretary of the Department of Environment and
Natural Resources to cease and desist from implementing Department of Environment and Natural
Resources Circular No. 2, series of 1998;
(4) The issuance of a writ of prohibition directing the Secretary of Budget and Management to cease
and desist from disbursing public funds for the implementation of the assailed provisions of R.A. 8371;
and
(5) The issuance of a writ of mandamus commanding the Secretary of Environment and Natural
Resources to comply with his duty of carrying out the States constitutional mandate to control and
supervise the exploration, development, utilization and conservation of Philippine natural resources. [7]
After due deliberation on the petition, the members of the Court voted as follows:
Seven (7) voted to dismiss the petition. Justice Kapunan filed an opinion, which the Chief Justice and
Justices Bellosillo, Quisumbing, and Santiago join, sustaining the validity of the challenged provisions of
R.A. 8371. Justice Puno also filed a separate opinion sustaining all challenged provisions of the law with
the exception of Section 1, Part II, Rule III of NCIP Administrative Order No. 1, series of 1998, the Rules
and Regulations Implementing the IPRA, and Section 57 of the IPRA which he contends should be
interpreted as dealing with the large-scale exploitation of natural resources and should be read in
conjunction with Section 2, Article XII of the 1987 Constitution. On the other hand, Justice Mendoza

voted to dismiss the petition solely on the ground that it does not raise a justiciable controversy and
petitioners do not have standing to question the constitutionality of R.A. 8371.
Seven (7) other members of the Court voted to grant the petition. Justice Panganiban filed a separate
opinion expressing the view that Sections 3 (a)(b), 5, 6, 7 (a)(b), 8, and related provisions of R.A. 8371
are unconstitutional. He reserves judgment on the constitutionality of Sections 58, 59, 65, and 66 of
the law, which he believes must await the filing of specific cases by those whose rights may have been
violated by the IPRA. Justice Vitug also filed a separate opinion expressing the view that Sections 3(a),
7, and 57 of R.A. 8371 are unconstitutional.Justices Melo, Pardo, Buena, Gonzaga-Reyes, and De Leon
join in the separate opinions of Justices Panganiban and Vitug.
As the votes were equally divided (7 to 7) and the necessary majority was not obtained, the case was
redeliberated upon. However, after redeliberation, the voting remained the same.Accordingly, pursuant
to Rule 56, Section 7 of the Rules of Civil Procedure, the petition is DISMISSED.
Attached hereto and made integral parts thereof are the separate opinions of Justices Puno, Vitug,
Kapunan, Mendoza, and Panganiban.
SO ORDERED.
G.R. No. L-12691

February 27, 1959

SIMEON T. DAGDAG, plaintiff-appellee,


vs.
VICENTE NEPOMUCENO, ET AL., defendants-appellants.
Jesus Paredes for appellee.
Mariano Sta. Romana for appellants.
BENGZON, J.:
Forwarded by the Court of Appeals, this lawsuit coming from Nueva Ecija, concern a small parcel of
land. Submitted for decision below upon a stipulation of facts, it raises legal questions only.
A portion of Lot No. 3786, Cabanatuan Cadaster (admittedly alienable or disposable public land way
back in 1916) is covered by Sales Patent No. 251 issued to Margarita Juanson, and also by lease No. 49
executed by the Bureau of Lands in favor of Andres de Vera. The overlapping was recently discovered,
and their successors in interest now litigate for possession and/or ownership.
The Sales Patent was inscribed in the office of the Register of Deeds on July 11, 1927, and Original
Certificate of Title No. 68 was accordingly issued in the same of Margarita Juanson, who later sold the
land of Remegio Juanson Bautista (1928), who in turn sold it to Balarian Incorporated (1929). In May
1950, Simeon T. Dagdag bought it from Balarin, Inc. After every sale, the corresponding Transfer
Certificate of Title was given out.
On the other hand, the lease to De Vera signed in June 1916 covered adjoining land of a bigger area. It
was transferred by him to Regino Nepomuceno. Originally for a 25-years period expiring on June 30,
1941, it was extended for another like period in 1949. Dagdag's title, and those of his predecessors
contained no annotation of such lease, of which neither he nor they any knowledge.
After purchasing the land, Simeon T. Dagdag had it relocated and the portion in question turned out to
be in possession of the heirs of Regino Nepomuceno, appellants herein allegedly by virtue of the lease.
The latter refused to surrender it, even in the face of Dagdag's patent and title, and despite the
Director of Lands' administrative determination in February 1953, practically holding that their contract
of lease did not, could not and should not extend to the area granted to Dagdag's predecessors.

Hence, this judicial proceeding instituted by Dagdag in the Nueva Ecija court of first instance, wherein
he was declared to be the owner of the whole Lot 3786 and entitled to the products thereof. The
Honorable Jose N. Leutrio, Judge, explained that "the sales patent issued in the name of Margarita
Juanson having been registered with the office of the Register of Deeds, and title having been issued
by the Register of Deeds in the name of Margarita Juanson, Lot 3786 was thereafter brought under the
operation of the Land Registration Act. The title issued in the name of Margarita Juanson, Original
Certificate of Title No. 68 was free from all liens and incumbrances. This land was transferred
successively, until it was acquired by the plaintiff herein, and the certificate of title was issued in his
name free from any lien or encumbrances, and free from the claim of Regino Nepomuceno as losses.
The plaintiffs herein cannot, therefore, be bound by the fact that Lot 3786 is within the lease of Andres
de Vera which had been transferred to Regino Nepomuceno, the father and predecessor of the
defendants herein. The said lease not having been annotated on the certificate of title, and it not
having been neither proved or alleged that the plaintiff had purchased the land knowing that Lot 3786
is a portion of the land leased to Andres de Vera which had been acquired by the defendant's
predecessors-in-interest, it cannot prejudice the plaintiff who is presumed to be an innocent purchaser
for value. The fact that the lease in favor of Andres de Vera had been registered, cannot bind and
prejudice the plaintiff for Lot 3786 being a registered land, he need not go farther than the title."
The above observations deserve our approval. They conform with our decisions on indefeasibility of
public land patents when registered in the corresponding Register of Deeds Office. 1 We regard these to
be veritable Torrens Title subject to no encumbrances except those stated therein, plus those specified
by the status (lease is not one of them).
In addition to the above reason given by his Honor, it should be remembered that when the lease was
renewed in 1949, the portion in question was no longer public land subject to the disposition of the
Director of Lands because it had already been granted to Margarita Juanson and had
become private property; therefore, it could not have been included in the renewal of such lease of
public land.
Defendant's position may be summed up, in their own words, as follows:
When the contract of lease of the predecessor of the defendants was duly issued and registered in the
office of the register of deeds of Nueva Ecija, and when the patent for the certificate of sale in favor of
the predecessor of the plaintiff was issued and registered in the said register of deeds of Nueva Ecija,
both documents have the force and effect of registered properties under the land Registration Act as
provided for in . . . (section 122 of the Land Registration Law). . . .
As the titles of the parties have come under the operation of the Land Registration Act, and in case of
overlapping titles, the older title should prevail. The title of the defendants was issued and registered
on June 14, 1916. The title of the plaintiff was registered on August 5, 1927. The title of the
defendants should, therefore prevail, and they should have been declared the owners of the land in
question. (pp. 8-9 Appellants Brief) [Emphasis Ours].
The flaw in their argument lies in the assumption that their lease contract constituted a "title", or deed
or conveyance within the meaning of section 122, which for convenience is quoted below:
Whenever public lands in the Philippine Islands belonging to the Government of the United States or
the Government of the Philippine Islands are alienated, granted, or conveyed to persons or to public or
private corporations, the same shall be brought forthwith under the operation of this Act and shall
become registered lands. It shall be the duty of the officials issuing the instrument of alienation, grant,
or conveyance in behalf of the Government to cause such instrument, before its delivery to the
grantee, to be filed with the register of deeds for the province where the land lies and to be there
registered like other deeds and conveyance, whereupon a certificate shall be entered as in other cases
of registered land, and an owner's duplicate issue to the grantee. The deed, grant, or instrument of
conveyance from the Government to the grantee shall not take effect as a conveyance or bind the

land, . . . After due registration and issue of the certificate and owner's duplicate such shall be
registered land for all purposes under this Act.
Upon carefully reading the above, we think it clear that the documents mentioned, wherein lands are
"alienated, granted, or conveyed", are documents transferring ownership not documents of lease,
transferring mere possession. Observe especially that the statue directs the issuance to the grantee of
"an owner's duplicate certificate". Appellants may not, therefore, assert a title just as good-so they
claim-as appellee's and older besides. So, the Torrens Title of appellee must prevail.
Judgment affirmed, with costs against appellants.
Paras, C.J., Padilla, Montemayor, Reyes, A., Bautista Angelo, Labrador, Concepcion and Endencia,
JJ., concur.

CHAVEZ V. PUBLIC ESTATE AUTHORITY

FACTS:
From the time of Marcos until Estrada, portions of Manila Bay were being reclaimed. A law was passed
creating the Public Estate Authority which was granted with the power to transfer reclaimed lands. Now
in this case, PEA entered into a Joint Venture Agreement with AMARI, a private corporation. Under the
Joint Venture Agreement between AMARI and PEA, several hectares of reclaimed lands comprising the
Freedom Islands and several portions of submerged areas of Manila Bay were going to be transferred
to AMARI .
ISSUE:
Whether or not the stipulations in the Amended JVA for the transfer to AMARI of lands, reclaimed or to
be reclaimed, violate the Constitution

RULING: YES!

Under the Public Land Act (CA 141, as amended), reclaimed lands are classified as alienable and
disposable lands of the public domain Section 3 of the Constitution: Alienable lands of the public
domain shall be limited to agricultural lands. Private corporations or associations may not hold such
alienable lands of the public domain except by lease The 157.84 hectares of reclaimed lands
comprising the Freedom Islands, now covered by certificates of title in the name of PEA, are alienable
lands of the public domain. PEA may lease these lands to private corporations but may not sell or
transfer ownership of these lands to private corporations. PEA may only sell these lands to Philippine
citizens, subject to the ownership limitations in the 1987 Constitution and existing laws. Clearly, the
Amended JVA violates glaringly Sections 2 and 3, Article XII of the 1987 Constitution. Under Article
1409 of the Civil Code, contracts whose object or purpose is contrary to law, or whose object is
outside the commerce of men, are inexistent and void from the beginning. The Court must perform
its duty to defend and uphold the Constitution, and therefore declares the Amended JVA null and void
ab initio.
CHAVEZ V. PUBLIC ESTATES AUTHORITY
384 SCRA 152

FACTS:
President Marcos through a presidential decree created PEA, which was tasked with the
development, improvement, and acquisition, lease, and sale of all kinds of lands. The then
president also transferred to PEA the foreshore and offshore lands of Manila Bay under the ManilaCavite Coastal
Road and Reclamation Project.
Thereafter, PEA was granted patent to the reclaimed areas of land and then, years later, PEA
entered into a JVA with AMARI for the development of the Freedom Islands. These two entered
into a joint venture in the absence of any public bidding.
Later, a privilege speech was given by Senator President Maceda denouncing the JVA as the
grandmother of all scams. An investigation was conducted and it was concluded that the lands that
PEA was conveying to AMARI were lands of the public domain; the certificates of title over the
Freedom Islands were void; and the JVA itself was illegal. This prompted Ramos to form an
investigatory committee on the legality of the JVA.
Petitioner now comes and contends that the government stands to lose billions by the
conveyance or sale of the reclaimed areas to AMARI. He also asked for the full disclosure of the
renegotiations happening between the parties.

ISSUE:
W/N stipulations in the amended JVA for the transfer to AMARI of the lands, reclaimed or to be
reclaimed, violate the Constitution.

HELD:
The ownership of lands reclaimed from foreshore and submerged areas is rooted in the Regalian
doctrine, which holds that the State owns all lands and waters of the public domain.
The 1987 Constitution recognizes the Regalian doctrine. It declares that all natural resources are
owned by the State and except for alienable agricultural lands of the public domain, natural
resources cannot be alienated.
The Amended JVA covers a reclamation area of 750 hectares. Only 157.84 hectares of the 750 hectare
reclamation project have been reclaimed, and the rest of the area are still submerged areas forming
part of Manila Bay. Further, it is provided that AMARI will reimburse the actual costs in
reclaiming the areas of land and it will shoulder the other reclamation costs to be incurred.
The foreshore and submerged areas of Manila Bay are part of the lands of the public domain, waters
and other natural resources and consequently owned by the State. As such, foreshore and
submerged areas shall not be alienable unless they are classified as agricultural lands of the
public domain. The mere reclamation of these areas by the PEA doesnt convert these inalienable
natural resources of the State into alienable and disposable lands of the public domain. There
must be a law or presidential

proclamation officially classifying these reclaimed lands as alienable and disposable if the law
has reserved them for some public or quasi-public use.

G.R. No. 134209

January 24, 2006

REPUBLIC OF THE PHILIPPINES, Petitioner,


vs.
CELESTINA NAGUIAT, Respondent.
DECISION
GARCIA, J.:
Before the Court is this petition for review under Rule 45 of the Rules of Court seeking the reversal of
the Decision1 dated May 29, 1998 of the Court of Appeals (CA) in CA-G.R. CV No. 37001 which affirmed
an earlier decision2 of the Regional Trial Court at Iba, Zambales, Branch 69 in Land Registration Case
No. N-25-1.
The decision under review recites the factual backdrop, as follows:
This is an application for registration of title to four (4) parcels of land located in Panan, Botolan,
Zambales, more particularly described in the amended application filed by Celestina Naguiat on 29
December 1989 with the Regional Trial Court of Zambales, Branch 69. Applicant [herein respondent]
alleges, inter alia, that she is the owner of the said parcels of land having acquired them by purchase
from the LID Corporation which likewise acquired the same from Demetria Calderon, Josefina Moraga
and Fausto Monje and their predecessors-in-interest who have been in possession thereof for more
than thirty (30) years; and that to the best of her knowledge, said lots suffer no mortgage or
encumbrance of whatever kind nor is there any person having any interest, legal or equitable, or in
possession thereof.
On 29 June 1990, the Republic of the Philippines [herein petitioner]. . . filed an opposition to the
application on the ground that neither the applicant nor her predecessors-in interest have been in
open, continuous, exclusive and notorious possession and occupation of the lands in question since 12
June 1945 or prior thereto; that the muniments of title and tax payment receipts of applicant do not
constitute competent and sufficient evidence of a bona-fide acquisition of the lands applied for or of
his open, continuous, exclusive and notorious possession and occupation thereof in the concept of (an)
owner; that the applicants claim of ownership in fee simple on the basis of Spanish title or grant can
no longer be availed of . . .; and that the parcels of land applied for are part of the public domain
belonging to the Republic of the Philippines not subject to private appropriation.
On 15 October 1990, the lower court issued an order of general default as against the whole world,
with the exception of the Office of the Solicitor General, and proceeded with the hearing of this
registration case.
After she had presented and formally offered her evidence . . . applicant rested her case. The Solicitor
General, thru the Provincial Prosecutor, interposed no objection to the admission of the exhibits. Later .
. . the Provincial Prosecutor manifest (sic) that the Government had no evidence to adduce. 3
In a decision4 dated September 30, 1991, the trial court rendered judgment for herein respondent
Celestina Naguiat, adjudicating unto her the parcels of land in question and decreeing the registration
thereof in her name, thus:
WHEREFORE, premises considered, this Court hereby adjudicates the parcels of land situated in Panan,
Botolan, Zambales, appearing on Plan AP-03-003447 containing an area of 3,131 square meters,

appearing on Plan AP-03-003446 containing an area of 15,322 containing an area of 15,387 square
meters to herein applicant Celestina T. Naguiat, of legal age, Filipino citizen, married to Rommel
Naguiat and a resident of Angeles City, Pampanga together with all the improvements existing thereon
and orders and decrees registration in her name in accordance with Act No. 496, Commonwealth Act
No. 14, [should be 141] as amended, and Presidential Decree No. 1529. This adjudication, however, is
subject to the various easements/reservations provided for under pertinent laws, presidential decrees
and/or presidential letters of instructions which should be annotated/ projected on the title to be
issued. And once this decision becomes final, let the corresponding decree of registration be
immediately issued. (Words in bracket added)
With its motion for reconsideration having been denied by the trial court, petitioner Republic went on
appeal to the CA in CA-G.R. CV No. 37001.
As stated at the outset hereof, the CA, in the herein assailed decision of May 29, 1998, affirmed that of
the trial court, to wit:
WHEREFORE, premises considered, the decision appealed from is hereby AFFIRMED.
SO ORDERED.
Hence, the Republics present recourse on its basic submission that the CAs decision "is not in
accordance with law, jurisprudence and the evidence, since respondent has not established with the
required evidence her title in fee simple or imperfect title in respect of the subject lots which would
warrant their registration under (P.D. 1529 or Public Land Act (C.A.) 141." In particular, petitioner
Republic faults the appellate court on its finding respecting the length of respondents occupation of
the property subject of her application for registration and for not considering the fact that she has not
established that the lands in question have been declassified from forest or timber zone to alienable
and disposable property.
Public forest lands or forest reserves, unless declassified and released by positive act of the
Government so that they may form part of the disposable agricultural lands of the public domain, are
not capable of private appropriation.5 As to these assets, the rules on confirmation of imperfect title do
not apply.6 Given this postulate, the principal issue to be addressed turns on the question of whether or
not the areas in question have ceased to have the status of forest or other inalienable lands of the
public domain.
Forests, in the context of both the Public Land Act7 and the Constitution8 classifying lands of the public
domain into "agricultural, forest or timber, mineral lands and national parks," do not necessarily refer
to a large tract of wooded land or an expanse covered by dense growth of trees and underbrush. As we
stated in Heirs of Amunategui 9A forested area classified as forest land of the public domain does not lose such classification simply
because loggers or settlers have stripped it of its forest cover. Parcels of land classified as forest land
may actually be covered with grass or planted to crops by kaingin cultivators or other farmers. "Forest
lands" do not have to be on mountains or in out of the way places. xxx. The classification is merely
descriptive of its legal nature or status and does not have to be descriptive of what the land actually
looks like. xxx
Under Section 2, Article XII of the Constitution,10 which embodies the Regalian doctrine, all lands of the
public domain belong to the State the source of any asserted right to ownership of land. 11 All lands
not appearing to be clearly of private dominion presumptively belong to the State. 12 Accordingly, public
lands not shown to have been reclassified or released as alienable agricultural land or alienated to a
private person by the State remain part of the inalienable public domain. 13 Under Section 6 of the
Public Land Act, the prerogative of classifying or reclassifying lands of the public domain, i.e., from
forest or mineral to agricultural and vice versa, belongs to the Executive Branch of the government

and not the court.14 Needless to stress, the onus to overturn, by incontrovertible evidence, the
presumption that the land subject of an application for registration is alienable or disposable rests with
the applicant.15
In the present case, the CA assumed that the lands in question are already alienable and disposable.
Wrote the appellate court:
The theory of [petitioner] that the properties in question are lands of the public domain cannot be
sustained as it is directly against the above doctrine. Said doctrine is a reaffirmation of the principle
established in the earlier cases . . . that open, exclusive and undisputed possession of alienable public
land for period prescribed by law creates the legal fiction whereby the land, upon completion of the
requisite period, ipso jure and without the need of judicial or other sanction, ceases to be public land
and becomes private property . (Word in bracket and underscoring added.)
The principal reason for the appellate courts disposition, finding a registerable title for respondent, is
her and her predecessor-in-interests open, continuous and exclusive occupation of the subject
property for more than 30 years. Prescinding from its above assumption and finding, the appellate
court went on to conclude, citing Director of Lands vs. Intermediate Appellate Court (IAC)16 and Herico
vs. DAR,17 among other cases, that, upon the completion of the requisite period of possession, the
lands in question cease to be public land and become private property.
Director of Lands, Herico and the other cases cited by the CA are not, however, winning cards for the
respondent, for the simple reason that, in said cases, the disposable and alienable nature of the land
sought to be registered was established, or, at least, not put in issue. And there lies the difference.
Here, respondent never presented the required certification from the proper government agency or
official proclamation reclassifying the land applied for as alienable and disposable. Matters of land
classification or reclassification cannot be assumed. It calls for proof. 18 Aside from tax receipts,
respondent submitted in evidence the survey map and technical descriptions of the lands, which,
needless to state, provided no information respecting the classification of the property. As the Court
has held, however, these documents are not sufficient to overcome the presumption that the land
sought to be registered forms part of the public domain. 19
It cannot be overemphasized that unwarranted appropriation of public lands has been a notorious
practice resorted to in land registration cases.20 For this reason, the Court has made it a point to stress,
when appropriate, that declassification of forest and mineral lands, as the case may be, and their
conversion into alienable and disposable lands need an express and positive act from the
government.21
The foregoing considered, the issue of whether or not respondent and her predecessor-in-interest have
been in open, exclusive and continuous possession of the parcels of land in question is now of little
moment. For, unclassified land, as here, cannot be acquired by adverse occupation or possession;
occupation thereof in the concept of owner, however long, cannot ripen into private ownership and be
registered as title.22
WHEREFORE, the instant petition is GRANTED and the assailed decision dated May 29, 1998 of the
Court of Appeals in CA-G.R. CV No. 37001 is REVERSED and SET ASIDE. Accordingly, respondents
application for original registration of title in Land Registration Case No. N-25-1 of the Regional Trial
Court at Iba, Zambales, Branch 69, is DENIED.
No costs.
G.R. No. L-43938 April 15, 1988

REPUBLIC OF THE PHILIPPINES (DIRECTOR OF FOREST DEVELOPMENT), petitioner,


vs.
HON. COURT OF APPEALS (THIRD DIVISION) and JOSE Y. DE LA ROSA, respondents.
CRUZ, J.:
The Regalian doctrine reserves to the State all natural wealth that may be found in the bowels of the
earth even if the land where the discovery is made be private. 1 In the cases at bar, which have been
consolidated because they pose a common issue, this doctrine was not correctly applied.
These cases arose from the application for registration of a parcel of land filed on February 11, 1965,
by Jose de la Rosa on his own behalf and on behalf of his three children, Victoria, Benjamin and
Eduardo. The land, situated in Tuding, Itogon, Benguet Province, was divided into 9 lots and covered by
plan Psu-225009. According to the application, Lots 1-5 were sold to Jose de la Rosa and Lots 6-9 to his
children by Mamaya Balbalio and Jaime Alberto, respectively, in 1964. 2
The application was separately opposed by Benguet Consolidated, Inc. as to Lots 1-5, Atok Big Wedge
Corporation, as to Portions of Lots 1-5 and all of Lots 6-9, and by the Republic of the Philippines,
through the Bureau of Forestry Development, as to lots 1-9. 3
In support of the application, both Balbalio and Alberto testified that they had acquired the subject
land by virtue of prescription Balbalio claimed to have received Lots 1-5 from her father shortly after
the Liberation. She testified she was born in the land, which was possessed by her parents under claim
of ownership. 4 Alberto said he received Lots 6-9 in 1961 from his mother, Bella Alberto, who declared
that the land was planted by Jaime and his predecessors-in-interest to bananas, avocado, nangka and
camote, and was enclosed with a barbed-wire fence. She was corroborated by Felix Marcos, 67 years
old at the time, who recalled the earlier possession of the land by Alberto's father. 5 Balbalio presented
her tax declaration in 1956 and the realty tax receipts from that year to 1964, 6 Alberto his tax
declaration in 1961 and the realty tax receipts from that year to 1964. 7
Benguet opposed on the ground that the June Bug mineral claim covering Lots 1-5 was sold to it on
September 22, 1934, by the successors-in-interest of James Kelly, who located the claim in September
1909 and recorded it on October 14, 1909. From the date of its purchase, Benguet had been in actual,
continuous and exclusive possession of the land in concept of owner, as evidenced by its construction
of adits, its affidavits of annual assessment, its geological mappings, geological samplings and trench
side cuts, and its payment of taxes on the land. 8
For its part, Atok alleged that a portion of Lots 1-5 and all of Lots 6-9 were covered by the Emma and
Fredia mineral claims located by Harrison and Reynolds on December 25, 1930, and recorded on
January 2, 1931, in the office of the mining recorder of Baguio. These claims were purchased from
these locators on November 2, 1931, by Atok, which has since then been in open, continuous and
exclusive possession of the said lots as evidenced by its annual assessment work on the claims, such
as the boring of tunnels, and its payment of annual taxes thereon. 9
The location of the mineral claims was made in accordance with Section 21 of the Philippine Bill of
1902 which provided that:
SEC. 21. All valuable mineral deposits in public lands in the philippine Islands both surveyed and
unsurveyed are hereby declared to be free and open to exploration, occupation and purchase and the
land in which they are found to occupation and purchase by the citizens of the United States, or of said
islands.
The Bureau of Forestry Development also interposed its objection, arguing that the land sought to be
registered was covered by the Central Cordillera Forest Reserve under Proclamation No. 217 dated

February 16, 1929. Moreover, by reason of its nature, it was not subject to alienation under the
Constitutions of 1935 and 1973. 10
The trial court * denied the application, holding that the applicants had failed to prove their claim of
possession and ownership of the land sought to be registered. 11 The applicants appealed to the
respondent court, * which reversed the trial court and recognized the claims of the applicant, but
subject to the rights of Benguet and Atok respecting their mining claims. 12 In other words, the Court of
Appeals affirmed the surface rights of the de la Rosas over the land while at the same time reserving
the sub-surface rights of Benguet and Atok by virtue of their mining claims.
Both Benguet and Atok have appealed to this Court, invoking their superior right of ownership. The
Republic has filed its own petition for review and reiterates its argument that neither the private
respondents nor the two mining companies have any valid claim to the land because it is not alienable
and registerable.
It is true that the subject property was considered forest land and included in the Central Cordillera
Forest Reserve, but this did not impair the rights already vested in Benguet and Atok at that time. The
Court of Appeals correctly declared that:
There is no question that the 9 lots applied for are within the June Bug mineral claims of Benguet and
the "Fredia and Emma" mineral claims of Atok. The June Bug mineral claim of plaintiff Benguet was one
of the 16 mining claims of James E. Kelly, American and mining locator. He filed his declaration of the
location of the June Bug mineral and the same was recorded in the Mining Recorder's Office on October
14, 1909. All of the Kelly claims ha subsequently been acquired by Benguet Consolidated, Inc.
Benguet's evidence is that it had made improvements on the June Bug mineral claim consisting of
mine tunnels prior to 1935. It had submitted the required affidavit of annual assessment. After World
War II, Benguet introduced improvements on mineral claim June Bug, and also conducted geological
mappings, geological sampling and trench side cuts. In 1948, Benguet redeclared the "June Bug" for
taxation and had religiously paid the taxes.
The Emma and Fredia claims were two of the several claims of Harrison registered in 1931, and which
Atok representatives acquired. Portions of Lots 1 to 5 and all of Lots 6 to 9 are within the Emma and
Fredia mineral claims of Atok Big Wedge Mining Company.
The June Bug mineral claim of Benguet and the Fredia and Emma mineral claims of Atok having been
perfected prior to the approval of the Constitution of the Philippines of 1935, they were removed from
the public domain and had become private properties of Benguet and Atok.
It is not disputed that the location of the mining claim under consideration was perfected prior to
November 15, 1935, when the Government of the Commonwealth was inaugurated; and according to
the laws existing at that time, as construed and applied by this court in McDaniel v. Apacible and
Cuisia (42 Phil. 749), a valid location of a mining claim segregated the area from the public domain.
Said the court in that case: The moment the locator discovered a valuable mineral deposit on the lands
located, and perfected his location in accordance with law, the power of the United States Government
to deprive him of the exclusive right to the possession and enjoyment of the located claim was gone,
the lands had become mineral lands and they were exempted from lands that could be granted to any
other person. The reservations of public lands cannot be made so as to include prior mineral perfected
locations; and, of course, if a valid mining location is made upon public lands afterwards included in a
reservation, such inclusion or reservation does not affect the validity of the former location. By such
location and perfection, the land located is segregated from the public domain even as against the
Government. (Union Oil Co. v. Smith, 249 U.S. 337; Van Mess v. Roonet, 160 Cal. 131; 27 Cyc. 546).
"The legal effect of a valid location of a mining claim is not only to segregate the area from the public
domain, but to grant to the locator the beneficial ownership of the claim and the right to a patent
therefor upon compliance with the terms and conditions prescribed by law. Where there is a valid

location of a mining claim, the area becomes segregated from the public domain and the property of
the locator." (St. Louis Mining & Milling Co. v. Montana Mining Co., 171 U.S. 650; 655; 43 Law ed., 320,
322.) "When a location of a mining claim is perfected it has the effect of a grant by the United States
of the right of present and exclusive possession, with the right to the exclusive enjoyment of all the
surface ground as well as of all the minerals within the lines of the claim, except as limited by the
extralateral right of adjoining locators; and this is the locator's right before as well as after the
issuance of the patent. While a lode locator acquires a vested property right by virtue of his location
made in compliance with the mining laws, the fee remains in the government until patent issues."(18
R.C.L. 1152) (Gold Creek Mining Corporation v. Hon. Eulogio Rodriguez, Sec. of Agriculture and
Commerce, and Quirico Abadilla, Director of the Bureau of Mines, 66 Phil. 259, 265-266)
It is of no importance whether Benguet and Atok had secured a patent for as held in the Gold Creek
Mining Corp. Case, for all physical purposes of ownership, the owner is not required to secure a patent
as long as he complies with the provisions of the mining laws; his possessory right, for all practical
purposes of ownership, is as good as though secured by patent.
We agree likewise with the oppositors that having complied with all the requirements of the mining
laws, the claims were removed from the public domain, and not even the government of the
Philippines can take away this right from them. The reason is obvious. Having become the private
properties of the oppositors, they cannot be deprived thereof without due process of law. 13
Such rights were not affected either by the stricture in the Commonwealth Constitution against the
alienation of all lands of the public domain except those agricultural in nature for this was made
subject to existing rights. Thus, in its Article XIII, Section 1, it was categorically provided that:
SEC. 1. All agricultural, timber and mineral lands of the public domain, waters, minerals, coal,
petroleum and other mineral oils, all forces of potential energy and other natural resources of the
Philipppines belong to the State, and their disposition, exploitation, development, or utilization shall be
limited to citizens of the Philippines or to corporations or associations at least 60% of the capital of
which is owned by such citizens, subject to any existing right, grant, lease or concession at the time of
the inauguration of the government established under this Constitution. Natural resources with the
exception of public agricultural lands, shall not be alienated, and no license, concession, or lease for
the exploitation, development or utilization of any of the natural resources shall be granted for a period
exceeding 25 years, except as to water rights for irrigation, water supply, fisheries, or industrial uses
other than the development of water power, in which case beneficial use may be the measure and the
limit of the grant.
Implementing this provision, Act No. 4268, approved on November 8, 1935, declared:
Any provision of existing laws, executive order, proclamation to the contrary notwithstanding, all
locations of mining claim made prior to February 8, 1935 within lands set apart as forest reserve under
Sec. 1826 of the Revised Administrative Code which would be valid and subsisting location except to
the existence of said reserve are hereby declared to be valid and subsisting locations as of the date of
their respective locations.
The perfection of the mining claim converted the property to mineral land and under the laws then in
force removed it from the public domain. 14 By such act, the locators acquired exclusive rights over the
land, against even the government, without need of any further act such as the purchase of the land or
the obtention of a patent over it. 15 As the land had become the private property of the locators, they
had the right to transfer the same, as they did, to Benguet and Atok.
It is true, as the Court of Appeals observed, that such private property was subject to the "vicissitudes
of ownership," or even to forfeiture by non-user or abandonment or, as the private respondents aver,
by acquisitive prescription. However, the method invoked by the de la Rosas is not available in the
case at bar, for two reasons.

First, the trial court found that the evidence of open, continuous, adverse and exclusive possession
submitted by the applicants was insufficient to support their claim of ownership. They themselves had
acquired the land only in 1964 and applied for its registration in 1965, relying on the earlier alleged
possession of their predecessors-in-interest. 16 The trial judge, who had the opportunity to consider the
evidence first-hand and observe the demeanor of the witnesses and test their credibility was not
convinced. We defer to his judgment in the absence of a showing that it was reached with grave abuse
of discretion or without sufficient basis. 17
Second, even if it be assumed that the predecessors-in-interest of the de la Rosas had really been in
possession of the subject property, their possession was not in the concept of owner of the mining
claim but of the property as agricultural land, which it was not. The property was mineral land, and
they were claiming it as agricultural land. They were not disputing the lights of the mining locators nor
were they seeking to oust them as such and to replace them in the mining of the land. In fact, Balbalio
testified that she was aware of the diggings being undertaken "down below" 18 but she did not mind,
much less protest, the same although she claimed to be the owner of the said land.
The Court of Appeals justified this by saying there is "no conflict of interest" between the owners of the
surface rights and the owners of the sub-surface rights. This is rather doctrine, for it is a well-known
principle that the owner of piece of land has rights not only to its surface but also to everything
underneath and the airspace above it up to a reasonable height. 19 Under the aforesaid ruling, the land
is classified as mineral underneath and agricultural on the surface, subject to separate claims of title.
This is also difficult to understand, especially in its practical application.
Under the theory of the respondent court, the surface owner will be planting on the land while the
mining locator will be boring tunnels underneath. The farmer cannot dig a well because he may
interfere with the operations below and the miner cannot blast a tunnel lest he destroy the crops
above. How deep can the farmer, and how high can the miner, go without encroaching on each other's
rights? Where is the dividing line between the surface and the sub-surface rights?
The Court feels that the rights over the land are indivisible and that the land itself cannot be half
agricultural and half mineral. The classification must be categorical; the land must be either
completely mineral or completely agricultural. In the instant case, as already observed, the land which
was originally classified as forest land ceased to be so and became mineral and completely mineral
once the mining claims were perfected. 20 As long as mining operations were being undertaken
thereon, or underneath, it did not cease to be so and become agricultural, even if only partly so,
because it was enclosed with a fence and was cultivated by those who were unlawfully occupying the
surface.
What must have misled the respondent court is Commonwealth Act No. 137, providing as follows:
Sec. 3. All mineral lands of the public domain and minerals belong to the State, and their disposition,
exploitation, development or utilization, shall be limited to citizens of the Philippines, or to
corporations, or associations, at least 60% of the capital of which is owned by such citizens, subject to
any existing right, grant, lease or concession at the time of the inauguration of government
established under the Constitution.
SEC. 4. The ownership of, and the right to the use of land for agricultural, industrial, commercial,
residential, or for any purpose other than mining does not include the ownership of, nor the right to
extract or utilize, the minerals which may be found on or under the surface.
SEC. 5. The ownership of, and the right to extract and utilize, the minerals included within all areas for
which public agricultural land patents are granted are excluded and excepted from all such patents.
SEC. 6. The ownership of, and the right to extract and utilize, the minerals included within all areas for
which Torrens titles are granted are excluded and excepted from all such titles.

This is an application of the Regalian doctrine which, as its name implies, is intended for the benefit of
the State, not of private persons. The rule simply reserves to the State all minerals that may be found
in public and even private land devoted to "agricultural, industrial, commercial, residential or (for) any
purpose other than mining." Thus, if a person is the owner of agricultural land in which minerals are
discovered, his ownership of such land does not give him the right to extract or utilize the said
minerals without the permission of the State to which such minerals belong.
The flaw in the reasoning of the respondent court is in supposing that the rights over the land could be
used for both mining and non-mining purposes simultaneously. The correct interpretation is that once
minerals are discovered in the land, whatever the use to which it is being devoted at the time, such
use may be discontinued by the State to enable it to extract the minerals therein in the exercise of its
sovereign prerogative. The land is thus converted to mineral land and may not be used by any private
party, including the registered owner thereof, for any other purpose that will impede the mining
operations to be undertaken therein, For the loss sustained by such owner, he is of course entitled to
just compensation under the Mining Laws or in appropriate expropriation proceedings. 21
Our holding is that Benguet and Atok have exclusive rights to the property in question by virtue of their
respective mining claims which they validly acquired before the Constitution of 1935 prohibited the
alienation of all lands of the public domain except agricultural lands, subject to vested rights existing
at the time of its adoption. The land was not and could not have been transferred to the private
respondents by virtue of acquisitive prescription, nor could its use be shared simultaneously by them
and the mining companies for agricultural and mineral purposes.
WHEREFORE, the decision of the respondent court dated April 30, 1976, is SET ASIDE and that of the
trial court dated March 11, 1969, is REINSTATED, without any pronouncement as to costs.
SO ORDERED.
Republic vs Leonor Dec. 23, 2009
DECISION
NACHURA, J.:

This is a petition for review on certiorari of the Court of Appeals (CA) Decision[1] dated December 19,
2003. The assailed decision adjudged the cancellation of the free patents and original certificates of
title (OCTs) over two of the five lots in question in favor of petitioner.
The antecedents of the case are as follows:
On December 16, 1991, petitioner Republic of the Philippines, represented by the Regional Executive
Director, Department of Environment and Natural Resources (DENR), Region IV, through the Office of
the Solicitor General, filed separate complaints for Cancellation of Free Patent and OCT and Reversion
against respondents Ignacio Leonor and Catalino Razon. The complaints involved the following
properties:

1.
In Civil Case No. 55-91: Free Patent No. (IV-3A)-2182, covered by OCT No. P-1676 in the name
of Ignacio Leonor, over Lot No. 10108, Cad. 511, Lemery Cadastre with an area of 722 square meters;

2.
In Civil Case No. 56-91: Free Patent No. (IV-3A)-2181, covered by OCT No. P-1675 in the name
of Ignacio Leonor, over Lot No. 8617, Cad. 511, Lemery Cadastre with an area of 706 square meters;
3.
In Civil Case No. 57-91: Free Patent No. (IV-3A)-2180, covered by OCT No. P-1674 in the name
of Catalino Razon, over Lot No.10109, Cad. 511, Lemery Cadastre, with an area of 722 square meters;
4.
In Civil Case No. 58-91: Free Patent No. (IV-3A)-1891, covered by OCT No. P-1127 in the name
of Ignacio Leonor, over Lot No. 9398, Cad. 511, Lemery Cadastre with an area of 2,066 square meters;
5.
In Civil Case No. 59-91: Free Patent No. (IV-3A)-1892, covered by OCT No. P-1128 in the name
of Catalino Razon, over Lot No. 9675, Cad. 511, Lemery Cadastre with an area of 1,944 square meters.
[2]

In Civil Case Nos. 55-91,[3] 56-91[4] and 57-91,[5] the complaints averred that, in an investigation
conducted by DENR-Region IV, it was ascertained that Lot Nos. 10108, 8617 and 10109 were part of
the non-disposable foreshore land and did not appear in the cadastral map or in the cadastral records
as having been officially surveyed by the DENR. These defects allegedly constituted fraud which, in
effect, ipso facto cancelled the free patents and the corresponding OCTs.
In contrast, the complaints in Civil Case Nos. 58-91 [6] and 59-91[7] alleged that, on the basis of a protest
filed by Luisa Ilagan Vda. de Agoncillo who claimed to be in possession of Lot Nos. 9398 and 9675
since time immemorial, an investigation was conducted by the DENR wherein it was discovered that
(1) although the said lots appeared in the cadastral map, they were not cadastrally surveyed or
approved cadastral lots as evidenced by the Alphabetical and Numerical List of Claimants; (2) the lots
were verified to be part of the early survey conducted on June 22, 1977 and identified as Lot No. 6192
of Cadastre 511, Lemery Cadastre, subsequently covered by Plan SWO-4A-000306-D in the name of
Luisa Ilagan; and (3) Lot Nos. 9398 and 9675 were conveyed to respondents, respectively, through an
Affidavit of Relinquishment of Rights executed on November 27, 1986 by a certain Anacleto Serwelas
who had no right whatsoever over the land. The complaints further averred that serious discrepancies
existed among the technical descriptions appearing in the certificates of title, the cadastral map and
the transfer of rights. These defects, according to the complaint, also constituted fraud which, in
effect,ipso facto cancelled the said patents and the corresponding OCTs.
On February 10, 1992, respondents filed their separate answers [8] uniformly stating as follows: (1) the
free patents were issued in accordance with existing law and procedure; (2) the subject lots were
surveyed by Geodetic Engineer Alexander Jacob of the Bureau of Lands and inspected and certified to
be alienable and disposable by the Land Inspector of the Bureau of Lands; (3) the right of action for the
cancellation of the same had already prescribed since more than one year had already lapsed since
the free patents were issued; (4) they had been in continuous, exclusive and notorious possession and
occupation of the lots for more than 30 years and they had developed them into a beach resort, with
valuable facilities; and (5) the subject lots were not investigated by the DENR-Region IV and there was
no resolution issued by the said office to that effect.

Luisa Ilagan was allowed to intervene in Civil Case Nos. 58-91 and 59-91. She claimed that Lot Nos.
9398 and 9675 were part of the parcel of land that she owned, designated as Lot No. 6192, Cad-511-D
of the Lemery Cadastre and covered by Tax Declaration No. 0527; that this parcel of land was surveyed
on June 22, 1977 and Plan SWO-4A-000306-D was approved on April 18, 1980; that she had been in
peaceful possession of the subject land for more than 60 years but, because of old age, she failed to
visit and supervise the land; that Anacleto Serwelas was her tenant who took advantage of her
absence and succeeded in selling the western portion of the subject land in favor of respondents,
without her knowledge and consent; and that in 1987, she learned of respondents applications for free
patent and of the issuance of the OCTs in their names; hence, she filed a formal protest with the DENR
asking for an investigation.[9]

In answer to these allegations, respondents averred that Luisa Ilagan had already sold her properties
to her tenants, and that Plan SWO-4A-000304 in her name was rejected by the Bureau of Lands as
shown in the Cadastral Map of Lemery Cadastre, Cad. 511, Case 22. [10] Luisa Ilagan replied that the
rejection of Plan SWO-4A-000304 was null and void for lack of notice. She insisted that respondents
had no right over the subject lots since they acquired them from Anacleto Serwelas, who was not the
owner of the properties.
On June 14, 2000, the Regional Trial Court rendered a decision in favor of respondents, thus:
WHEREFORE, for insufficiency of evidence presented by the plaintiff Republic of the Philippines and the
Intervenor, to prove that fraud was committed to acquire the title of the land in dispute, all the above
five entitled cases are hereby ordered DISMISSED for lack of merit.
IT IS SO ORDERED.[11]
The heirs of Luisa Ilagan and the petitioner filed separate appeals with the CA.
On February 11, 2002, the CA partially granted petitioners prayers. It declared that two of the five
lotsLot Nos. 10108 and 10109are foreshore lands. The CA noted that (a) serious discrepancies exist
between the cadastral map and the technical description in the OCTs covering these two lots; (b) the
said lots do not appear in the cadastral map; (c) Atty. Raymundo L. Apuhin, petitioners witness,
testified that the said lots were not surveyed and approved by the DENR; and (d) they do not appear to
be covered by corresponding tax declarations. Based on the foregoing, the CA concluded that these
two lots are foreshore lands. Consequently, it ordered the cancellation of Free Patent No. (IV-3A)-2182
and OCT No. P-1676 over Lot No. 10108 and Free Patent No. (IV-3A)-2180 and OCT No. P-1674
over Lot No. 10109. As for Lot Nos. 8617, 9398 and 9675, the CA sustained the trial courts finding that
there was no sufficient evidence to prove that they are foreshore lands or part of Luisa Ilagans
property. The dispositive portion of the decision reads:
WHEREFORE, premises considered, the assailed decision dated June 14, 2000 of the RTC, Branch 5,
Lemery, Batangas in Civil Cases Nos. 55-91 to 59-91 is hereby AFFIRMED with MODIFICATION. Free
Patent No. (IV-3A)-2182 with the corresponding OCT No. P-1676 in the name of Ignacio Leonor over Lot
No. 10108, and Free Patent No. (IV-3A)-2180 with the corresponding OCT No. P-1674, in the name of
Catalino Razon over Lot [No.] 10109 are hereby ordered CANCELLED from the Registry of Deeds of
Batangas.
The rest of the decision stands.
SO ORDERED.[12]
This petition for review on certiorari seeks the reversion of Lot Nos. 8617, 9398 and 9675 to petitioner.
On this score, petitioner ascribes the following error to the appellate court:
THE COURT OF APPEALS COMMITTED SERIOUS ERROR OF LAW WHEN IT SUSTAINED THE VALIDITY OF
THE THREE SUBJECT FREE PATENTS AND TITLES ALBEIT THEY PERTAIN TO INALIENABLE FORESHORE
LANDS AND DESPITE THE FRAUDULENT ENTRIES IN RESPONDENTS FREE PATENT APPLICATIONS. [13]
Petitioner argues that the lands are inalienable foreshore lands. It points out that the five lots comprise
the whole Leonor Beach Resort and that when the technical descriptions of the subject lots were
plotted on the cadastral map of Barangay Nonong Castro, the lots were identified as foreshore lands,
which are not capable of appropriation. [14] Petitioner adds that the burden is on respondents to prove
that the lands that have been registered in their names are alienable and disposable. [15]
Petitioner further contends that, assuming that the subject lands are not foreshore lands, the free
patents should nonetheless be cancelled, because respondents committed fraud and made

misrepresentations in their free patent applications in that (a) they declared that the subject lots were
cadastrally surveyed when, in truth, they do not appear in the approved Cadastral Plan of Lemery,
Batangas, Cad. 511, Case 22; (b) respondent Ignacio Leonor declared that he acquired Lot No. 9398
from Moises and Ricardo Peren and Vicente de Roxas, whose names do not however appear on the lists
of claimants for Barangay Nonong Castro, Case 22, Lemery Cadastre, indicating that they are fictitious
persons; (c) respondent Ignacio Leonor failed to enter the names of his predecessors-in-interest as to
Lot No. 8617, as required in the free patent application; (d) serious discrepancies were noted in the
description of Lot No. 9398 in the application for free patent and in the technical description in OCT No.
P-1127; and (e) Lot No. 9675 does not appear in the lists of claimants. [16]
Incidentally, it should be pointed out that, other than Lot Nos. 10108 and 10109, only Lot No. 8617 was
alleged in the complaint (Civil Case No. 59-91) to be part of the indisposable foreshore land. In fact,
there is no piece of evidence pointing to Lot Nos. 9398 and 9675 as being foreshore lands. Petitioner
seeks the cancellation of the free patents over Lot Nos. 9398 and 9675 solely on the ground that they
were procured through fraud and misrepresentation.

The Court finds that the petition has no merit


As a rule, the findings of fact of the trial court when affirmed by the CA are final and conclusive on, and
cannot be reviewed on appeal by, this Court as long as they are borne out by the records or are based
on substantial evidence. The Court is not a trier of facts, its jurisdiction being limited to reviewing only
errors of law that may have been committed by the lower courts. [17] But to appease any doubt on the
correctness of the assailed ruling, we have carefully perused the records and, nonetheless, arrived at
the same conclusion.
To be sure, petitioner was not able to adequately establish that Lot No. 8617 is a foreshore land or that
the free patents covering Lot Nos. 8617, 9398 and 9675 were procured through fraud or
misrepresentation.
At the outset, petitioner argues that the burden to prove that the lands in question are alienable and
disposable is upon respondents. The argument is out of line. This case is not a land registration
proceeding but involves reversion of lands already registered in the names of respondents. At this
stage, it would be reasonable to presume that respondents had established that the properties are
alienable and disposable considering that they have already succeeded in obtaining free patents and
OCTs over the properties. In this reversion proceeding, premised on the claim that the property is
foreshore land or that the patents were obtained through fraud or misrepresentation, the burden is
now upon petitioner to prove such allegations.

With regard to Lot No. 8617, the records reveal that the only piece of evidence alluding to this lot being
foreshore land is the testimony of Atty. Apuhin of the DENR-Region IV, which is quoted as follows:
QAnd what did you find in relation to the Free Patent No. (IV-3A) 2180 insofar as that
plotting made by the Legal Division of the DENR is concerned?
A-

In my request, I found out that x x x Lot No. 8617 is a foreshore lot.

Q-

How about the survey record, what did you find insofar as Lot No. 8617 is concerned?

AIn verification with the Survey Division, Lot No. 8617 is definitely a part of [the] foreshore
lot as shown in the approved cadastral map of Lemery.

Q-

By the way, when you speak of foreshore lot, what do you mean?

AIt is an area covered by the flow of tide in its highest equational tide which is 20 meters
from the highest equational tide.

Q-

If it is a foreshore land, can it be the subject of Free Patent application?

A-

No, sir.

xxxx
QAfter conducting the necessary investigation insofar as Lot 8617 is concerned, what is
your conclusion?
AAfter conducting the necessary investigation insofar as Lot 8617 is concerned, Lot 8617 is
a foreshore lot.
Q-

If it is a foreshore lot, what is your conclusion?

A-

A foreshore lot cannot be the subject of acquisition [of] Free Patent.

QIf it cannot be the subject of acquisition, what is the effect on the application for Free
Patent of x x Lot No. 8617?
A-

The application for x x x Free Patent should not have been approved. [18]

Certainly, Atty. Apuhins testimony fails to convince us. The interview markedly lacks details as to how
he conducted an investigation to determine whether Lot No. 8617 is foreshore land or an explanation
as to how he arrived at his conclusion. Although it was stated in the records that Atty. Apuhin
conducted an ocular inspection, his only finding on the basis of this inspection was that the lots had
already been developed as a beach resort. In his direct testimony, he vaguely stated that the lot is
foreshore land as shown in the cadastral map. We have examined the said cadastral map of Barangay
Nonong Castro, Lemery, Batangas but we noticed that it does not indicate, in any way, that Lot No.
8617 is foreshore land. What is obvious in the said map is that the lot is close to the waters
of Balayan Bay. However, the lands proximity alone to the waters alone does not necessarily make it a
foreshore land.[19] It must be shown that the land is between high and low water and left dry by the flux
and reflux of the tides or between the high and low water marks, which is alternatively wet and dry
according to the flow of the tide.[20]
We likewise agree with the CA that petitioner was not able to establish that fraud or misrepresentation
attended the application for free patents. In the same way that petitioner has the burden of proving
that Lot No. 8617 is a foreshore land, petitioner, as the party alleging that fraud and misrepresentation
vitiated the application for free patents, also bears the burden of proof. [21] Fraud and misrepresentation
are never presumed, but must be proved by clear and convincing evidence; mere preponderance of
evidence is not even adequate.[22]
To show that there was fraud, petitioner insists that the three lots do not appear in the cadastral map
of Barangay Nonong Castro, which allegedly indicates that they were not cadastrally surveyed. This is
manifestly untrue as the said cadastral map of Lemery Cadastre, Cad. 511, Case 22, clearly includes
and indicates the locations of Lot Nos. 8617, 9398 and 9675.

Petitioner also cites discrepancies in the description of Lot No. 9398 in the free patent application and
in the technical description in OCT No. P-1127. If true, such discrepancies would not necessarily imply
that respondents employed fraud or misrepresentation in obtaining the free patent. After all, there was
no proof that the lot referred to in the free patent application was different from the lot described in
OCT No. P-1127.
Further, petitioner points out that in the free patent application for Lot No. 8617, respondent Ignacio
Leonor did not indicate the names of his predecessors-in-interest and the date when he began his
possession and cultivation of the lot. Petitioner contends that this was in violation of Section 91[23] of
the Public Land Act and, as such, resulted in the ipso facto cancellation of the free patent.
The mere omission of an information from the patent application, though essential, does not, per se,
cause the ipso facto cancellation of the patent. It must be shown that the information withheld would
have resulted in the disapproval of the free patent application had it been disclosed. The names of the
predecessors-in-interest are obviously required to be indicated in the application form in order to show
that the applicant has complied with the occupation and cultivation requirement under the law. In this
case, petitioner had no evidence showing that respondents had not complied with the occupation and
cultivation requirement under the law. Considering this, we are ill-equipped to pronounce the ipso
facto cancellation of free patents.
With regard to Lot No. 9398, petitioner argues that the names of the predecessors of Ignacio Leonor
Vicente de Roxas, Moises and Ricardo Peren listed in the application form for the free patent are
fictitious persons, as their names do not appear in the List of Claimants. The list also does not
include Lot No. 9675, which petitioner claims is an indication that the lot was not cadastrally surveyed.
Again, we are not convinced. Undoubtedly, the list of claimants is evidence that the lots enumerated
therein were cadastrally surveyed, and that the name indicated after each lot number was that of the
claimant of the lot at the time of the survey. But despite Atty. Apuhins testimony, the list cannot be
given weight particularly with respect to lots not included therein. In other words, the list cannot be
taken as evidence that lots that were not included in the list were not cadastrally surveyed or that only
the claimants named therein had rights over that particular lot. This is only reasonable considering
that it is not even known, for sure, when the list was made, how it was prepared, and how often it was
updated.
Atty. Apuhins testimony on the preparation of the list and on there being no other list for other lots in
Barangay Nonong Castro is not worthy of credence. He admitted during trial that he was not privy to
the preparation of the list. Apparently, he was also not the actual custodian of the list since a certain
Florencio V. Carreon, Chief, Records Unit, certified the copy of the list. Atty. Apuhins ignorance on this
matter is made more apparent by the following testimony:

Q-

You were a privy in the preparation of the list?

A-

I am not.

QSince you were not a privy to the preparation of the list[,] you must have inquired how
often was the list prepared[.] The list given to you was the list on the approved Cadastral Survey?
A-

Case No. 22.

Court:
QThe question of the Court is that, constantly this list is revised because of the approval of
certain claimants in relation to the approved survey?

A-

Yes, sir.

QAs you inquired, what was the date when this list shown to you alphabetically was
approved?
AQ-

It was approved on March 12, 1987.


Because the alphabetical list is based on that?

A-

Yes, sir.

xxxx
QDo you know, based on record when the cadastral survey was implemented, that cadastral
survey which was approved on March 12, 1987?
A-

I did not see.

Q-

This list of claimants, they are listed while the cadastral survey is being done?

A-

Yes, sir.

Q- Based on the approved survey plan?


A-

Yes, sir.

Q- It would appear at the time of the survey?


A-

Yes, sir.

Q- The survey plan is approved later on?


A-

Yes, sir.

Q- On your own knowledge based on the investigation, was there any list subsequent to March 12,
1987 released by the DENR?
A-

None, sir.

Q- But you look[ed] at this application on your investigation?


A-

Yes, sir.

Q- And you affirm that this was the last list of claimants alphabetically done in relation to [Barangay]
Nonong Castro?
A-

Yes, sir.

Fiscal:
QA-

Aside from this list of claimants, has there been other list of claimants, prior list?
I have no knowledge[24]

We also do not believe that Moises and Ricardo Peren and Vicente de Roxas are fictitious
persons. From the investigation conducted by Atty. Apuhin, he learned that Moises Peren executed a
Waiver of Real Rights on June 16, 1986 in favor of Ignacio Leonor. [25] It also appears from the records

that these persons were respondents in a case for accion reivindicatoria and quieting of title filed by
Luisa Ilagan.[26]
From the foregoing, the dearth of petitioners evidence is glaring. DENR-Region IV did not conduct a
thorough investigation of the alleged irregularities imputed to respondents in obtaining the free
patents. There was not even a written report on the investigation submitted to the court. In view of
this, we are constrained to sustain the findings of both the trial court and the appellate court and to
deny the petition.
WHEREFORE, premises considered, the petition is DENIED. The Court of Appeals Decision
dated December 19, 2003 is AFFIRMED.

Almagro vs Kwan Oct. 20, 2010


This is a consolidation of two separate petitions for review, [1] assailing the 4 April 2006 Decision[2] and
the 31 October 2006 Resolution[3] of the Court of Appeals in CA-G.R. SP Nos. 71237 and 71437.
This case involves Lot No. 6278-M, a 17,181 square meter parcel of land covered by TCT No. T-11397.
Lot No. 6278-M is located at Maslog, Sibulan, Negros Oriental and is registered in the name of spouses
Kwan Chin and Zosima Sarana. Respondents are the legitimate children of spouses Kwan Chin and
Zosima Sarana, who both died intestate on 2 November 1986 and 23 January 1976, respectively, in
Dumaguete City. Upon the death of their parents, respondents inherited Lot No. 6278-M through
hereditary succession.

On 18 September 1996, respondents filed with the Municipal Trial Court (MTC) an action for recovery of
possession and damages against spouses Rogelio and Lourdes Duran, spouses Romulo Vinalver and
Elsa Vinalver,[4] spouses Marte[5] Bati-on and Liz E. Bati-on, spouses Pablo Deciar and Marlyn Deciar,
spouses Salvador Palongpalong and Bienvenida Palongpalong, spouses Sabas Kiskis and Eduvigis
Kiskis, spouses Pio Tubat, Jr. and Encarnita Tubat, spouses Andres Tubat and Leonides Tubat, spouses
George Tubat and Noela Tubat, spouses Dodong Go and Alice Go, spouses Delano Bangay and Maria
Bangay,[6] spouses Simeon Pachoro and Margarita Pachoro, spouses Cepriano[7]Tubat and Elsa Tubat,
spouses Jovito Remolano and Editha Orlina Remolano, spouses Nelson Miravalles and Erlene Miravalles,
Dronica Orlina,[8] Clarita Barot Lara, Conchita Orlina, Antonia Malahay and the Philippine National Police
(PNP),[9] Agan-an, Sibulan, Negros Oriental. Subsequently, spouses Manuel Almagro and Elizabeth
Almagro intervened as successors-in-interest of spouses Delano Bangay and Maria Bangay.

During pre-trial, the parties agreed to refer the case to the Chief of the Land Management Services
Division, PENRO-DENR, Dumaguete City, to conduct a verification survey ofLot No. 6278-M. When the
PENRO personnel failed to conduct the verification survey, the court and the parties designated
Geodetic Engineer Jorge Suasin, Sr. (Engr. Suasin) as joint commissioner to do the task. Engr. Suasin
conducted the verification and relocation survey of Lot No. 6278-M on 12-13 September 2000 in the
presence of the parties, some of their lawyers, and the MTC Clerk of Court. Thereafter, Engr. Suasin
submitted a written report with the following findings:

WRITTEN REPORT

Comes now, the undersigned Geodetic Engineer Jorge S. Suasin, Sr., to this Honorable Court, most
respectfully submit the following written report of the verification and relocation survey of the lot 6278M located at Maslog, Sibulan, Negros Oriental with T.C.T. No. T-11397 owned by Salvacion G. Kwan, et
al.

A. That a big portion of the lot is submerged under the sea and only a small portion remain as dry land.

B. That some of the defendants have constructed their buildings or houses inside the dry land while
others have constructed outside or only a small portion of their buildings or houses are on the said dry
land.

The defendants and their buildings or houses are as follows:

1. Sps. Rogelio Duran . . . . . . . . . . . . . . . . . . . . . . . . . inside


2. Sps. Romulo Vinalver. . . . . . . . . . . . . . . . . . . . . . . inside
3. Sps. Marto Bati-on . . . . . . . . . . . . . . . . . . . . . . . . . inside
4. Sps. Salvador Palongpalong . . . . . . . . . . . . . . . . . . inside
5. Sps. Pablo Deciar . . . . . . . . . . . . . . . . . . . . . . . . . . inside
6. Sps. Sabas Kiskis . . . . . . . . . . . . . . . . . . . . . . . . . . .inside
7. Sps. Pio Tubat, Jr. . . . . . . . . . . . . . . . . . . . . . . . 2 houses, the first house a portion, and the second
one - inside
8. Sps. Andres Tubat . . . . . . . . . . . . . . . . . . . . . . . . . . inside
9. Sps. George Tubat . . . . . . . . . . . . . . . . . . . . . . . . . . portion
10. Sps. Dodong Go . . . . . . . . . . . . . . . . . . . . . . . . . . inside
11. Sps. Delano Bangay-Almagro . . . . . . . . . . . . . . . . portion
12. Sps. Simeon Pachoro . . . . . . . . . . . . . . . . . . . . . . . inside
13. Sps. Cipriano Tubat . . . . . . . . . . . . . . . . . . . . . . . . inside
14. Sps. Jovito Remolano . .. . . . . . . . . . . . . . . . . . . . . inside
15. Sps. Nelson Miravalles . . . . . . . . . . . . . . . . . . . . . cottage and
house - outside
16. Monica Orlina . . . . . . . . . . . . . . . . . . . . . . . . . . . . cottage inside

and house portion


17. Clarita Barot . . . . . . . . . . . . . . . . . . . . . . . . . . . . . . outside
18. Conchita Orlina . . . . . . . . . . . . . . . . . . . . . . . . . . . outside
19. Antonia Malahay . . . . . . . . . . . . . . . . . . . . . . . . . . outside

The verification and relocation survey was executed last September 12-13, 2000 with the presence of
both parties and of the Clerk of Court. The cost of the survey was FIFTEEN THOUSAND
PESOS (P15,000) shouldered by the plaintiffs and the defendants equally.

Enclosed are a blue print of the sketch plan and a xerox copy of the land title of the said lot.

Respectfully submitted by:

(Sgd) JORGE SUASIN, SR.


Geodetic Engineer
After the court admitted Engr. Suasin's report and the pleadings of the parties, respondents filed a
motion for judgment on the pleadings, which the MTC granted.
In its Judgment dated 11 May 2001, the MTC dismissed the complaint on the ground that the remaining
dry portion of Lot No. 6278-M has become foreshore land and should be returned to the public domain.
The MTC explained:
The term foreshore refers to that part of the land adjacent to the sea which is alternately covered and
left dry by the ordinary flow of the tides. Foreshore lands refers to the strip of land that lies between
the high and low water marks and that is alternately wet and dry according to the flow of the tide. The
term foreshore land clearly does not include submerged lands.
From these definitions, it is safe to conclude that the remaining dry portion of Lot No. 6278-M is now
foreshore land. A big portion of the said lot is presently underwater or submerged under the sea. When
the sea moves towards the estate and the tide invades it, the invaded property becomes foreshore
land and passes to the realm of public domain. The subject land, being foreshore land, should
therefore be returned to the public domain. Besides, Article 420 of the Civil Code provides:
Art. 420. The following thin[g]s are property of public dominion:
(1) Those intended for public use, such as roads, canals, rivers, torrents, ports and bridges constructed
by the State, banks, shores, roadsteads, and others of similar character;

Plaintiff cannot use the doctrine of indefeasibility of their Torrens title, as property in question is clearly
foreshore land. At the time of its registration, property was along the shores. In fact, it is bounded by

the Taon Strait on the NW along lines 2-3-4. The property was of public dominion and should not have
been subject of registration. The survey showed that the sea had advanced and the waves
permanently invaded a big portion of the property making the land part of the shore or the beach. The
remaining dry land is foreshore and therefore should be returned to the public domain. [11]
Respondents appealed to the Regional Trial Court (RTC). The RTC conducted ocular inspections of Lot
No. 6278-M on two separate dates: on 5 October 2001 during low tide and on 15 October 2001 when
the high tide registered 1.5 meters. All the parties and their lawyers were notified before the two
ocular inspections were conducted. During the ocular inspections, in which some parties and their
lawyers were present, the RTC observed that the small portion referred to by Engr. Suasin as dry land
in his report actually remained dry even during high tide. [12] Thus, the RTC concluded that the disputed
remaining portion of Lot No. 6278-M is not foreshore land. The RTC stated:

It is the Court's considered view that the small portion of plaintiff's property which remains as dry land
is not within the scope of the well-settled definition of foreshore and foreshore land as mentioned
above. For one thing, the small dry portion is not adjacent to the sea as the term adjacent as
defined in Webster's Dictionary means contiguous or touching one another or lying next to. Secondly,
the small dry portion is not alternately wet and dry by the ordinary flow of the tides as it is dry land.
Granting, as posited by defendants, that at certain times of the year, said dry portion is reached by the
waves, then that is not anymore caused by the ordinary flow of the tide as contemplated in the above
definition. The Court then finds that the testimony of Engr. Suasin dovetails with the import and
meaning of foreshore and foreshore land as defined above.
Anent the case of Republic vs. Court of Appeals, 281 SCRA 639, also cited in the appealed judgment,
the same has a different factual milieu. Said case involves a holder of a free patent on a parcel of land
situated at Pinagtalleran, Caluag, Quezon who mortgaged and leased portions thereof within the
prescribed five-year period from the date of issuance of the patent. It was established in said case that
the land subject of the free patent is five (5) to six (6) feet deep under water during high tide and two
(2) feet deep at low tide. Such is not the situation of the remaining small dry portion which plaintiffs
seek to recover in the case at bar. [13]
On 8 January 2002, the RTC rendered its Decision, [14] the dispositive portion of which reads:
WHEREFORE, all told and circumspectly considered, the appealed judgment is hereby reversed and set
aside insofar as it states that plaintiffs are not entitled to recover possession of the property in
question.
Plaintiffs-appellants have the right to recover possession of the remaining small dry portion of the
subject property in question. It is further ordered to remand this case to the court of origin for the
reception of further evidence to determine who among the defendants-appellees are builders or
possessors in good faith and who are not and once determined, to apply accordingly the pertinent laws
and jurisprudence on the matter.
SO ORDERED.[15]
Petitioners moved for reconsideration, which the RTC denied in its Order [16] dated 6 May 2002.
Petitioners filed separate petitions for review with the Court of Appeals, alleging that the disputed
portion of Lot No. 6278-M is no longer private land but has become foreshore land and is now part of
the public domain.

The Ruling of the Court of Appeals

On 4 April 2006, the Court of Appeals promulgated its decision, affirming with modification the RTC
Decision. The dispositive portion of the Court of Appeals Decision [17]reads:
WHEREFORE, the instant petitions for review are DENIED. And the Decision dated January 8, 2002 of
Branch 38 of the Regional Trial Court of Dumaguete City is hereby AFFIRMED with
MODIFICATION as regards the dispositive portion only. Based on the written report of Geodetic Engr.
Suasin categorically indentifying who among herein petitioners are illegally occupying a portion of Lot
No. 6278-M, the following petitioners are ordered to vacate the premises and/or remove the
houses and/or cottages constructed on Lot No. 6278-M within thirty (30) days from finality
of judgment, namely: 1)Sps. Rogelio Duran, 2) Sps. Romulo Vinalver, 3) Sps. Marto Bati-on, 4) Sps.
Salvador Palongpalong, 5) Sps. Pablo Deciar, 6) Sps. Sabas Kiskis, 7) Sps. Pio Tubat, Jr. (first house
portion, second house inside), 8) Sps. Andres Tubat, 9) George Tubat (portion), 10) Sps. Dodong Go,
11) Sps. Delano Bangay-Almagro (portion), 12) Sps. Simeon Pachoro, 13) Sps. Cipriano Tubat, 14) Sps.
Jovito Remolano and 15) Monica Orlina (cottageinside and house portion).
Costs against petitioners.
SO ORDERED.
In modifying the RTC Decision, the Court of Appeals explained:
Lastly, the argument that the RTC decision was vague and indefinite is utterly bereft of merit. We have
found no reversible error in the appreciation of the facts and in the application of the law by the RTC
which will warrant the reversal of the questioned decision. However, litigation must end and terminate
sometime and somewhere, and it is essential to the administration of justice that the issues or causes
therein should be laid to rest. Hence, in keeping with this principle, We modify the assailed decision
insofar as the dispositive portion is concerned. It is our considered view that there is no longer a need
to determine who among the petitioners are builders in good faith or not considering that it has been
established in the MTC that they knew all along that the subject lot is a titled property. As such,
petitioners should vacate and/or demolish the houses and/or cottages they constructed on Lot No.
6278-M as stated in the written report of Geodetic Engineer Jorge S. Suasin, Sr. Remanding this case to
the court of origin would not only unduly prolong the resolution of the issues of this case, but would
also subject the parties to unnecessary expenses. [19]

Hence, these consolidated petitions.


The Issue

The primary issue in this case is whether the disputed portion of Lot No. 6278-M is still private land
or has become foreshore land which forms part of the public domain
The Ruling of the Court
We find the petitions without merit.
Petitioners contend that the disputed portion of Lot No. 6278-M is already foreshore land. In fact, most
of them allegedly have foreshore lease permits from the Department of Environment and Natural
Resources (DENR) on the said foreshore land.

However, petitioners failed to present evidence to prove their claim that they are holders of foreshore
lease permits from the DENR. Thus, the RTC Order dated 6 May 2002 stated:
Defendants-appellees have been harping that they have been granted foreshore leases by DENR.
However, this is merely lip service and not supported at all by concrete evidence. Not even an iota of
evidence was submitted to the lower court to show that defendants-appellees herein have been
granted foreshore leases.[20]

Although the MTC concluded that the subject land is foreshore land, we find such conclusion contrary
to the evidence on record.
It is undisputed that the subject land is part of Lot No. 6278-M, which is covered by TCT No. T-11397,
registered in the name of respondents' parents, Kwan Chin and Zosimo Sarana. In fact, as found by the
Court of Appeals, even the Provincial Environment and Natural Resources Officer (PENRO) declared in
May 1996 that Lot No. 6278-M is a private property covered by a Torrens Title and that petitioners
should vacate the disputed property or make other arrangements with respondents. [21]
Furthermore, from the report of Engr. Suasin, the geodetic engineer designated by the court and the
parties as joint commissioner to conduct the survey, it can be clearly gleaned that the contested land
is the small portion of dry land of Lot No. 6278-M. Even in his testimony, Engr. Suasin was adamant
in stating that the remaining portion of Lot No. 6278-M is not foreshore because it is already dry land
and is away from the shoreline.[22] Because of this apparent contradiction between the evidence and
the conclusion of the MTC, the RTC conducted ocular inspection twice, during low tide and high tide,
and observed that the disputed portion of Lot No. 6278-M actually remained dry land even during high
tide. Thus, the RTC concluded that the said land is not foreshore land. On appeal, the Court of Appeals
adopted the findings and conclusion of the RTC that the disputed land is not foreshore land and that it
remains as private land owned by respondents.
We are in accord with the conclusion of the Court of Appeals and the RTC that the disputed land is not
foreshore land. To qualify as foreshore land, it must be shown that the land lies between the high and
low water marks and is alternately wet and dry according to the flow of the tide. [23] The land's proximity
to the waters alone does not automatically make it a foreshore land. [24]
Thus, in Republic of the Philippines v. Lensico,[25] the Court held that although the two corners of the
subject lot adjoins the sea, the lot cannot be considered as foreshore land since it has not been proven
that the lot was covered by water during high tide.
Similarly in this case, it was clearly proven that the disputed land remained dry even during high tide.
Indeed, all the evidence supports the conclusion that the disputed portion of Lot No. 6278-M is not
foreshore land but remains private land owned by respondents.
WHEREFORE, we DENY the petitions. We AFFIRM the 4 April 2006 Decision and the 31 October 2006
Resolution of the Court of Appeals in CA-G.R. SP Nos. 71237 and 71437.
office of the city mayor of paranaque vs ebio June 23, 2010
VILLARAMA, JR., J.:

Before us is a petition for review on certiorari under Rule 45 of the 1997 Rules of Civil Procedure, as
amended, assailing the January 31, 2007 Decision[1] and June 8, 2007 Resolution[2] of the Court of
Appeals (CA) in CA-G.R. SP No. 91350 allegedly for being contrary to law and jurisprudence. The CA

had reversed the Order[3] of the Regional Trial Court (RTC) of Paraaque City, Branch 196, issued on
April 29, 2005 in Civil Case No. 05-0155.
Below are the facts.
Respondents claim that they are the absolute owners of a parcel of land consisting of 406 square
meters, more or less, located at 9781 Vitalez Compound in Barangay Vitalez, Paraaque City and
covered by Tax Declaration Nos. 01027 and 01472 in the name of respondent Mario D. Ebio. Said land
was an accretion of Cut-cut creek. Respondents assert that the original occupant and possessor of the
said parcel of land was their great grandfather, Jose Vitalez. Sometime in 1930, Jose gave the land to
his son, Pedro Vitalez. From then on, Pedro continuously and exclusively occupied and possessed the
said lot. In 1966, after executing an affidavit declaring possession and occupancy,[4] Pedro was able to
obtain a tax declaration over the said property in his name.[5] Since then, respondents have been
religiously paying real property taxes for the said property.[6]
Meanwhile, in 1961, respondent Mario Ebio married Pedros daughter, Zenaida. Upon Pedros advice, the
couple established their home on the said lot. In April 1964 and in October 1971, Mario Ebio secured
building permits from the Paraaque municipal office for the construction of their house within the said
compound.[7] On April 21, 1987, Pedro executed a notarized Transfer of Rights[8] ceding his claim over
the entire parcel of land in favor of Mario Ebio. Subsequently, the tax declarations under Pedros name
were cancelled and new ones were issued in Mario Ebios name.[9]
On March 30, 1999, the Office of the Sangguniang Barangay of Vitalez passed Resolution No. 08, series
of 1999[10] seeking assistance from the City Government of Paraaque for the construction of an
access road along Cut-cut Creek located in the said barangay. The proposed road, projected to be eight
(8) meters wide and sixty (60) meters long, will run from Urma Drive to the main road of Vitalez
Compound[11] traversing the lot occupied by the respondents. When the city government advised all
the affected residents to vacate the said area, respondents immediately registered their opposition
thereto. As a result, the road project was temporarily suspended.[12]
In January 2003, however, respondents were surprised when several officials from the barangay and
the city planning office proceeded to cut eight (8) coconut trees planted on the said lot. Respondents
filed letter-complaints before the Regional Director of the Bureau of Lands, the Department of Interior
and Local Government and the Office of the Vice Mayor.[13] On June 29, 2003, the Sangguniang
Barangay of Vitalez held a meeting to discuss the construction of the proposed road. In the said
meeting, respondents asserted their opposition to the proposed project and their claim of ownership
over the affected property.[14] On November 14, 2003, respondents attended another meeting with
officials from the city government, but no definite agreement was reached by and among the parties.
[15]
On March 28, 2005, City Administrator Noli Aldip sent a letter to the respondents ordering them to
vacate the area within the next thirty (30) days, or be physically evicted from the said property.[16]
Respondents sent a letter to the Office of the City Administrator asserting, in sum, their claim over the
subject property and expressing intent for a further dialogue.[17] The request remained unheeded.
Threatened of being evicted, respondents went to the RTC of Paraaque City on April 21, 2005 and
applied for a writ of preliminary injunction against petitioners.[18] In the course of the proceedings,
respondents admitted before the trial court that they have a pending application for the issuance of a
sales patent before the Department of Environment and Natural Resources (DENR).[19]
On April 29, 2005, the RTC issued an Order[20] denying the petition for lack of merit. The trial court
reasoned that respondents were not able to prove successfully that they have an established right to
the property since they have not instituted an action for confirmation of title and their application for
sales patent has not yet been granted. Additionally, they failed to implead the Republic of the
Philippines, which is an indispensable party.

Respondents moved for reconsideration, but the same was denied.[21]


Aggrieved, respondents elevated the matter to the Court of Appeals. On January 31, 2007, the Court of
Appeals issued its Decision in favor of the respondents. According to the Court of Appeals-The issue ultimately boils down to the question of ownership of the lands adjoining Cutcut Creek
particularly Road Lot No. 8 (hereinafter RL 8) and the accreted portion beside RL 8.
The evidentiary records of the instant case, shows that RL 8 containing an area of 291 square meters is
owned by Guaranteed Homes, Inc. covered by TCT No. S-62176. The same RL 8 appears to have been
donated by the Guaranteed Homes to the City Government of Paraaque on 22 March 1966 and which
was accepted by the then Mayor FLORENCIO BERNABE on 5 April 1966. There is no evidence however,
when RL 8 has been intended as a road lot.
On the other hand, the evidentiary records reveal that PEDRO VITALEZ possessed the accreted
property since 1930 per his Affidavit dated 21 March 1966 for the purpose of declaring the said
property for taxation purposes. The property then became the subject of Tax Declaration No. 20134
beginning the year 1967 and the real property taxes therefor had been paid for the years 1966, 1967,
1968, 1969, 1970, 1972, 1973, 1974, 1978, 1980, 1995, 1996, 1997, 1998, 1999, 2000, 2001, 2002,
2003, and 2004. Sometime in 1964 and 1971, construction permits were issued in favor of Appellant
MARIO EBIO for the subject property. On 21 April 1987, PEDRO VITALEZ transferred his rights in the
accreted property to MARIO EBIO and his successors-in-interest.
Applying [Article 457 of the Civil Code considering] the foregoing documentary evidence, it could be
concluded that Guaranteed Homes is the owner of the accreted property considering its ownership of
the adjoining RL 8 to which the accretion attached. However, this is without the application of the
provisions of the Civil Code on acquisitive prescription which is likewise applicable in the instant case.
xxxx
The subject of acquisitive prescription in the instant case is the accreted portion which [was] duly
proven by the Appellants. It is clear that since 1930, Appellants together with their predecessor-ininterest, PEDRO VITALEZ[,] have been in exclusive possession of the subject property and starting
1964 had introduced improvements thereon as evidenced by their construction permits. Thus, even by
extraordinary acquisitive prescription[,] Appellants have acquired ownership of the property in
question since 1930 even if the adjoining RL 8 was subsequently registered in the name of Guaranteed
Homes. x x x.
xxxx
Further, it was only in 1978 that Guaranteed Homes was able to have RL 8 registered in its name,
which is almost fifty years from the time PEDRO VITALEZ occupied the adjoining accreted property in
1930. x x x.
xxxx
We likewise note the continuous payment of real property taxes of Appellants which bolster their right
over the subject property. x x x.
xxxx
In sum, We are fully convinced and so hold that the Appellants [have] amply proven their right over the
property in question.
WHEREFORE, premises considered, the instant appeal is hereby GRANTED. The challenged Order of the
court a quo is REVERSED and SET ASIDE.

SO ORDERED.[22]
On June 8, 2007, the appellate court denied petitioners motion for reconsideration. Hence, this petition
raising the following assignment of errors:
I.
WHETHER OR NOT THE DECISION AND RESOLUTION OF THE HONORABLE COURT OF APPEALS
THAT RESPONDENTS HAVE A RIGHT IN ESSE IS IN ACCORD WITH THE LAW AND ESTABLISHED
JURISPRUDENCE[;]

II.
WHETHER OR NOT THE DECISION AND RESOLUTION OF THE HONORABLE COURT OF APPEALS
THAT THE SUBJECT LOT IS AVAILABLE FOR ACQUISITIVE PRESCRIPTION IS IN ACCORD WITH THE LAW
AND ESTABLISHED JURISPRUDENCE[;] AND

III. WHETHER OR NOT THE STATE IS AN INDISPENSABLE PARTY TO THE COMPLAINT FILED BY
RESPONDENTS IN THE LOWER COURT.[23]
The issues may be narrowed down into two (2): procedurally, whether the State is an indispensable
party to respondents action for prohibitory injunction; and substantively, whether the character of
respondents possession and occupation of the subject property entitles them to avail of the relief of
prohibitory injunction.
The petition is without merit.
An action for injunction is brought specifically to restrain or command the performance of an act.[24] It
is distinct from the ancillary remedy of preliminary injunction, which cannot exist except only as part or
as an incident to an independent action or proceeding. Moreover, in an action for injunction, the
auxiliary remedy of a preliminary prohibitory or mandatory injunction may issue.[25]
In the case at bar, respondents filed an action for injunction to prevent the local government of
Paraaque City from proceeding with the construction of an access road that will traverse through a
parcel of land which they claim is owned by them by virtue of acquisitive prescription.
Petitioners, however, argue that since the creek, being a tributary of the river, is classified as part of
the public domain, any land that may have formed along its banks through time should also be
considered as part of the public domain. And respondents should have included the State as it is an
indispensable party to the action.
We do not agree.
It is an uncontested fact that the subject land was formed from the alluvial deposits that have
gradually settled along the banks of Cut-cut creek. This being the case, the law that governs ownership
over the accreted portion is Article 84 of the Spanish Law of Waters of 1866, which remains in effect,
[26] in relation to Article 457 of the Civil Code.
Article 84 of the Spanish Law of Waters of 1866 specifically covers ownership over alluvial deposits
along the banks of a creek. It reads:
ART. 84. Accretions deposited gradually upon lands contiguous to creeks, streams, rivers, and lakes, by
accessions or sediments from the waters thereof, belong to the owners of such lands.[27]
Interestingly, Article 457 of the Civil Code states:

Art. 457. To the owners of lands adjoining the banks of rivers belong the accretion which they gradually
receive from the effects of the current of the waters.
It is therefore explicit from the foregoing provisions that alluvial deposits along the banks of a creek do
not form part of the public domain as the alluvial property automatically belongs to the owner of the
estate to which it may have been added. The only restriction provided for by law is that the owner of
the adjoining property must register the same under the Torrens system; otherwise, the alluvial
property may be subject to acquisition through prescription by third persons.[28]
In contrast, properties of public dominion cannot be acquired by prescription. No matter how long the
possession of the properties has been, there can be no prescription against the State regarding
property of public domain.[29] Even a city or municipality cannot acquire them by prescription as
against the State.[30]
Hence, while it is true that a creek is a property of public dominion,[31] the land which is formed by
the gradual and imperceptible accumulation of sediments along its banks does not form part of the
public domain by clear provision of law.
Moreover, an indispensable party is one whose interest in the controversy is such that a final decree
would necessarily affect his/her right, so that the court cannot proceed without their presence.[32] In
contrast, a necessary party is one whose presence in the proceedings is necessary to adjudicate the
whole controversy but whose interest is separable such that a final decree can be made in their
absence without affecting them.[33]
In the instant case, the action for prohibition seeks to enjoin the city government of Paraaque from
proceeding with its implementation of the road construction project. The State is neither a necessary
nor an indispensable party to an action where no positive act shall be required from it or where no
obligation shall be imposed upon it, such as in the case at bar. Neither would it be an indispensable
party if none of its properties shall be divested nor any of its rights infringed.
We also find that the character of possession and ownership by the respondents over the contested
land entitles them to the avails of the action.
A right in esse means a clear and unmistakable right.[34] A party seeking to avail of an injunctive relief
must prove that he or she possesses a right in esse or one that is actual or existing.[35] It should not
be contingent, abstract, or future rights, or one which may never arise.[36]
In the case at bar, respondents assert that their predecessor-in-interest, Pedro Vitalez, had occupied
and possessed the subject lot as early as 1930. In 1964, respondent Mario Ebio secured a permit from
the local government of Paraaque for the construction of their family dwelling on the said lot. In 1966,
Pedro executed an affidavit of possession and occupancy allowing him to declare the property in his
name for taxation purposes. Curiously, it was also in 1966 when Guaranteed Homes, Inc., the
registered owner of Road Lot No. 8 (RL 8) which adjoins the land occupied by the respondents, donated
RL 8 to the local government of Paraaque.
From these findings of fact by both the trial court and the Court of Appeals, only one conclusion can be
made: that for more than thirty (30) years, neither Guaranteed Homes, Inc. nor the local government
of Paraaque in its corporate or private capacity sought to register the accreted portion. Undoubtedly,
respondents are deemed to have acquired ownership over the subject property through prescription.
Respondents can assert such right despite the fact that they have yet to register their title over the
said lot. It must be remembered that the purpose of land registration is not the acquisition of lands,
but only the registration of title which the applicant already possessed over the land. Registration was
never intended as a means of acquiring ownership.[37] A decree of registration merely confirms, but
does not confer, ownership.[38]

Did the filing of a sales patent application by the respondents, which remains pending before the
DENR, estop them from filing an injunction suit?
We answer in the negative.
Confirmation of an imperfect title over a parcel of land may be done either through judicial
proceedings or through administrative process. In the instant case, respondents admitted that they
opted to confirm their title over the property administratively by filing an application for sales patent.
Respondents application for sales patent, however, should not be used to prejudice or derogate what
may be deemed as their vested right over the subject property. The sales patent application should
instead be considered as a mere superfluity particularly since ownership over the land, which they
seek to buy from the State, is already vested upon them by virtue of acquisitive prescription.
Moreover, the State does not have any authority to convey a property through the issuance of a grant
or a patent if the land is no longer a public land.[39]
Nemo dat quod dat non habet. No one can give what he does not have. Such principle is equally
applicable even against a sovereign entity that is the State.
WHEREFORE, the petition is DENIED for lack of merit. The January 31, 2007 Decision, as well as the July
8, 2007 Resolution, of the Court of Appeals in CA-G.R. SP No. 91350 are hereby AFFIRMED.
With costs against petitioners.
Acting Registrar of Land Titles and deeds of Pasay City vs RTC Makati April 26, 1990
SARMIENTO, J.:
The petitioners ** charge His Honor, Judge Francisco Velez, of the Regional Trial Court, Branch 57,
Makati, Metro Manila, with grave abuse of discretion in issuing an order authorizing the private
respondent, through Domingo Palomares, to perform acts of ownership over a 2,574-hectare parcel of
land known as Hacienda de Maricabanspread out in various parts of Makati, Pasig, Taguig, Pasay City,
and Paraaque. There is no controversy as to the facts.
On November 5, 1985, the private respondent, Domingo Palomares, as administrator of the heirs of
Delfin Casal, commenced suit with the Regional Trial Court, Branch 132, Makati, Metro Manila for
declaratory relief, quieting of title, cancellation of Transfer Certificate of Title No. 192, and cancellation
of entries upon Original Certificate of Title No. 291.
Palomares had earlier come to this Court (February 27, 1985) on a similar petition, and in addition, to
direct the Register of Deeds to issue a duplicate owner's copy of Original Certificate of Title No. 291,
embracing allegedly Hacienda de Maricaban, in lieu of the (alleged) lost one. On September 9, 1985,
the Court denied the petition for lack of merit. (G.R. No. 69834).
On December 19, 1985, the petitioners filed their answer.
On June 2, 1986, the private respondent filed a motion to admit amended complaint impleading the
Republic of the Philippines and the Registers of Deeds of Pasig, Makati, and Pasay City as partiesrespondents, and alleging, among other things, that: (1) on October 1, 1906, the Court of Land
Registration (James Ostrand, Presiding Judge) confirmed the title of Dolores Pascual Casal y Ochoa, a
native of Madrid, Spain, over the 2,574-hectare parcel above-mentioned; (2) on October 17, 1906, the
Register of Deeds of Rizal issued OCT No. 291 in her name; (3) upon her death, and successive deaths
of her heirs, the property devolved on Gerardo, Reynaldo, Lolita, and Erlinda, all surnamed Casal, great
grandchildren of Dolores; (4) no conveyances or dispositions of any kind have been allegedly made
upon the parcel; (5) TCT No. 192, which covers the same landholding, is allegedly spurious and
inexistent; (6) the State itself, by placing 27,213,255 square meters thereof under a military

reservation (Fort McKinley now Fort Bonifacio), by Proclamation No. 423, and fifty hectares thereof
pursuant to Proclamation No. 192, had been guilty of landgrabbing; (7) any and all holders of any and
all TCTs emanating therefrom or from TCT No. 192, are null, void, and of no force and effect; and (8) as
a consequence thereof, the heirs of Dolores Casal suffered various damages and attorney's fees.
On June 26, 1986, the petitioners filed an answer, stating, among other things, that: (1) the estate of
Dolores Casal (or Delfin Casal, her grandchild) is not a juridical person authorized by law to bring suit;
(2) the Registers of Deeds of Makati, Pasig, and Pasay City are not the real parties in interest, but
rather, the registered owners over which the court had not acquired jurisdiction; (3) the non-joinder of
the real parties in interest is fatal; (4) OCT No. 291 has long been cancelled; (5) Judge Gregorio Pineda
of the then Court of First Instance of Rizal, Branch XXI, Pasig, had earlier denied prayers for the
issuance of duplicate owner's copy of OCT No. 291 because the land embraced therein had been
validly delivered to the Government; (6) the Supreme Court itself had denied the Casals' appeal; *** (7)
as a consequence, res judicata is a bar; (8) prescription has also set in; and (9) the Casal's claims can
not validly override the titles of innocent purchasers for value.
On August 29, 1986, the respondent judge issued a temporary restraining order, directing the
petitioners to cease and desist from performing the acts complained of.
In a subsequent memorandum, the petitioners alleged that Dolores Casal had conveyed the property
to the Government of the United States in 1906 and the Manila Railroad Company on which Judge
Ostrand, the Presiding Judge of the Court of Land Registration, later Justice of this Court, had stamped
his imprimatur.
On October 12, 1987, the respondent court issued an order in the tenor, as follows:
No other opposition having been registered, this Court hereby resolves to grant the plaintiffs' prayer in
the OMNIBUS MOTION in order to safeguard the integrity of the land embraced in OCT 291, hereby
authorizing for this purpose the plaintiff Domingo C. Palomares:
1. To order such subdivision and/or individual survey or surveys within Parcel II, Parcel III and Parcel IV
under Survey Plan Psu-2031 by a licensed geodetic engineer or engineers at plaintiffs' expense in
order to facilitate and simplify the efficient administration of the property described in OCT 291; and
2. To sell, exchange, lease or otherwise dispose (of) any area or areas or portion or portions thereof,
subject to the approval of the Intestate Estate Court, to cover expenses for the payment of taxes to
which the property is subject, as well as expenses of administration and for the protection of the
integrity of the said lands.
SO ORDERED.

Eleven days later, or on October 23, 1987 to be precise, it issued another order, as follows:
Acting on the plaintiffs MOTION dated October 15, 1987 praying for the issuance of a Writ of Execution
implementing the Order of this Court dated October 12, 1987 before the expiration of the time to
appeal, and after inquiring from the plaintiff's counsel for their reason in seeking the same, the Court
hereby issues this clarificatory order affirming the power of the plaintiff Domingo C. Palomares to
execute and perform the acts authorized in the said Order of October 12, 1987 without the need of a
Writ of Execution, where no relief has been sought therefrom by any party, said Order being
implementable at the instance of the said plaintiff Domingo C. Palomares, anytime when the said Order
becomes final 15 days after the said plaintiff received copy of the same (see Section 39, Chapter IV,
B.P. Blg. 129). Plaintiff Domingo C. Palomares may therefore take whatever steps he considers
appropriate for the implementation of the said Order without need of further Orders or additional
authority from this Court.

SO ORDERED.

The petitioners filed a notice of appeal; the respondent court, however, denied it"
against . . . an interlocutory order. . . 4

"it being directed

Hence, this recourse.


The petitioners interpose the following questions:
A. Whether or not respondent Court can validly decide before trial in favor of private respondent the
ownership and possession of the 25,743,514 square meters (of) land known as "Hacienda de
Maricaban", which is the main issue in this case;
B. Whether or not respondent Court can validly allow private respondent to exercise and perform all
acts of ownership and possession over the said land before trial
C. Whether or not respondent Court has acquired jurisdiction to hear and decide this action;
D. Whether of not respondent Court committed grave abuse of discretion amounting to lack of
jurisdiction in not dismissing this action or allowing petitioners to appeal from the orders in question.

In their comment, the private respondent averred, among other things, that: (1) the respondent court,
contrary to the petitioners' claim, did not decide the case "before trial"; (2) OCT No. 291 had not been
validly cancelled and that the rubber stamp impression thereon, "CANCELLED" is a forgery; (3) the act
of Judge Pineda, in denying issuance of OCT No. 291, duplicate owner's copy, can not be
considered res judicata because that case involved purportedly a mere petition for issuance of
duplicate owner's copy; (4) non-joinder of proper parties is not a jurisdictional defect; (5) the TCTs
issued thereafter are a nullity because OCT No. 291 had not been shown to have been duly cancelled;
(6) OCT No. 291 has become imprescriptible; and (7) the private respondent has a valid right of
dominion over the property.
In the meantime, the private respondent came to this Court on certiorari (G.R. No. 90176) alleging that
on December 15, 1987, in connection with Sp. Proc. No. P-2993 of the Regional Trial Court, Branch 118,
Pasay City, entitled "In the matter of the Intestate Estate of the Late Fortunato Santiago and
Mariano Pantanilla Crisanta P. Santiago, et al., Petitioners," Judge Conrado Vasquez, Jr. issued an order
disposing of certain parcels which the private respondent claims as forming part and parcel
of Hacienda de Maricaban.
On June 20, 1988, the respondent judge in G.R. No. 81564 filed his own comment, asserting, among
other things, that: (1) what he had sought to bar, by virtue of injunction, was incursions and forcible
entries of trespassers and squatters; (2) the petitioners can not rightly claim that he had prematurely
adjudicated the case, because there was allegedly no decision to begin with; (3) that he issued the writ
of preliminary injunction in order only to maintain the status quo ante bellum that is, to re-place the
private respondent, which had been allegedly in prior possession, in possession; (4) he did not
allegedly authorize unbridled "acts of ownership" to be exercised on the property; (5) all rights of
dominion given thereon were subject to the approval of the intestate estate court; (6) he denied the
notice of appeal because the order dated October 12, 1987, was interlocutory in nature from which no
appeal lies; (7) as to jurisdiction, the various motions filed by petitioners, allegedly accepting the
court's jurisdiction, have clothed the court with jurisdiction, and that besides, the jurisdictional
question was never raised except now.
On July 7, 1988, the petitioners filed a reply traversing the respondent judge's allegations.
On August 26, 1988, the respondent judge filed a supplemental comment. He reiterated that the writ
of injunction was directed only on such spaces not occupied by the Government (Fort Bonifacio,

Libingan ng mga Bayani, Ninoy Aquino International Airport, Nayong Pilipino, Population Commission,
National Science and Development Board, and National Housing Authority).
Meanwhile, Atty. Antonio J. Dalangpan for and on behalf purportedly of the "Heirs of Delfin Casal" and
the private respondent, Domingo Palomares, file a Comment/Opposition in Intervention", dated
December 23, 1988 asking for the outright dismissal of the petition.
On December 14, 1989, the private respondent filed a manifestation, stating, among other things, that
assuming OCT No. 291 had been cancelled, there was still basis for the respondent judge to prevent
landgrabbers from entering into vacant portions of the state embraced thereby.
The Court finds the issues, quintessentially, to be:
(1) Is OCT No. 291 still valid and subsisting?
(2) Did the respondent judge, in issuing the orders, dated October 12 and October 23, 1987, commit a
grave abuse of discretion equivalent to lack or excess of jurisdiction?
I.
Is OCT No. 291 still valid and subsisting?
The Court takes judicial notice of the fact that the hectarage embraced by TCT No. 192 (OCT No. 291)
consists of Government property. Three things persuade the Court: (1) the decrees of Proclamations
Nos. 192 and 435; (2) the incontrovertible fact that OCT No. 291 has been duly cancelled; and (3) the
division of the Court of Appeals in AC-G.R. CV No. 00293, affirming the decision of Hon. Gregorio
Pineda, Judge of the then Court of First Instance of Rizal, Branch XXI, in LRC (GLRO) Rec. No. 2484,
Case No. R-1467 thereof, entitled "In Re: Issuance of Owner's Duplicate of Certificate of Title No. 291,"
as well as our own Resolution, in G.R. No. 69834, entitled "Domingo Palomares, et al., v. Intermediate
Appellate Court".
(a)
Proclamation No. 192 ("RESERVING FOR THE VETERANS CENTER SITE PURPOSES CERTAIN PARCEL OF
LAND OF THE PUBLIC DOMAIN SITUATED IN THE PROVINCE OF RIZAL, ISLAND OF LUZON") and
Proclamation No. 423 ("RESERVING FOR MILITARY PURPOSES CERTAIN PARCELS OF THE PUBLIC
DOMAIN SITUATED IN THE MUNICIPALITY OF PASIG, TAGUIG, AND PARAAQUE PROVINCE OF RIZAL,
AND PASAY CITY") have the character of official assertions of ownership, and the presumption is that
they have been issued by right of sovereignty and in the exercise of the State's dominical authority. We
take not only judicial notice thereof 6 but accept the same as a valid asseveration of regalian light over
property.
With respect to the premises occupied by the Libingan ng mga Bayani, Ninoy Aquino International
Airport, Nayong Pilipino, the Population Commission, National Science and Development Board, and
the National Housing Authority, we do not have the slightest doubt that they stand on Government
property by sheer presumption that, unless otherwise shown, what the Government occupies is what
the Government owns.
While there is no presumption that property is Government property until otherwise shown, because
the law recognizes private ownership, thus:
Art. 425. Property of private ownership, besides the patrimonial property of the State, provinces, cities,
and municipalities, consists of all property belonging to private persons, either individually or
collectively. 7

we find hard evidence on record that: (1) the property covered by OCT No. 291 had been conveyed to
the United States of America; (2) it had been later ceded to the Republic of the Philippines, and (3) as a
consequence, OCT No. 291 was cancelled upon final order of Judge Ostrand.
Be that as it may, the private respondent in G.R. No. 81564 is pressed hard to establish the fact that
portions of the property, especially the open spaces referred to in the lower court's writ of injunction
and the private respondent's manifestation of December 14, 1989, and which open spaces it claims to
be outside Maricaban, areindeed outside Maricaban (or OCT 291). With respect, however, to parts
thereof on which Fort Bonifacio, Libingan ng mga Bayani, Ninoy Aquino International Airport, Nayong
Pilipino, Population Commission National Science and Development Board, and National Housing
Authority sit, the hands of the private respondent are tied.
Claims that Judge Ostrand's decree was a counterfeit is not only self-serving, it finds no support from
the records. The presumptions is "that official duty has been regularly performed," 8 and the burden is
on the private respondent to prove irregular performance. The barren insistence that Judge Ostrands
order was a forgery is not sufficient to overthrow the presumption. To begin with, the act of forgery has
been seasonably disputed by the petitioners. Secondly, the Acting Registrar of Deeds of Pasig, who
supposedly certified to the fake character of Judge Ostrand's order, has himself joined the other
petitioners in opposing the reconveyance sought.
(b)
The decision in AC-G.R. No. 00293, dismissing the private respondent's petition for the issuance of a
new owner's copy of OCT No. 291, a dismissal affirmed by this Court in G.R. No. 69834, also militates
against the return of the property to the heirs of Delfin Casal. The Appellate Court's judgment, a
judgment sustained by this Court, operates as, at the very least, the law of the case between the
parties, that OCT No. 291 has been cancelled and the land covered has been conveyed and ceded to
the National Government. The fact that AC-G.R. CV No. 00293 dealt with a petition for issuance of lost
owner's duplicate copy is no argument because be that as it may, the private respondent can not
rightfully say that the heirs of Delfin Casal still have title to the land. If it can not secure a new owner's
copy, it can mean that they have lost title thereto.
(c)
The principle of res judicata is also a bar to the instant proceedings. It should be noted that in G.R. No.
69834, Mr. Domingo Palomares prayed:
WHEREFORE, premises considered it is most respectfully prayed to the most Honorable Supreme
Court, that in the name of law, justice and fair play, to prevent and frustrate "land-grabbing" by the
government, decision be rendered:
FIRST, That a thorough review of the aforementioned resolution of the Intermediate Appellate Court be
made;
SECOND, That after due consideration, the resolution subject of review be set aside based on the
aforestated assignment of error;
THIRD, That the Order of the Lower Court dated Jan. 19, 1977 be affirmed as the lawful and valid order;
FOURTH, To erase all doubts by declaring OCT No. 291 as continuously and existing validly against the
whole world;
FIFTH, Clearing OCT No. 291 of all adverse claims, since the herein petitioners are the true and legally
declared heirs; and

SIXTH, Ordering the Register of Deeds of Pasig, Rizal to issue the Owner's Duplicate Copy of OCT No.
291.
Petitioner-Appellant further prays for other just and equitable reliefs. ****
When we therefore denied that petition, we, in effect, held that reconstitution (of lost duplicate owner's
copy) was not possible because the mother title (OCT No. 291) had been duly cancelled. And when we
therefore declared OCT No. 291 to have been cancelled, we perished all doubts as to the invalidity of
Mr. Palomares' pretenses of title to Maricaban. Our judgment was conclusive not only as to Mr.
Palomares, but also as to the existing status of the property. As we have held:
The lower Court correctly ruled that the present action is barred by the final judgment rendered in the
previous case of Tuason & Co. vs. Aguila, Civil Case No. Q-4275, of the Court of First Instance of Rizal.
The reason is plain: if the herein appellants really had a preferential right to a conveyance of the land
from J.M. Tuason & Co., or if the certificate of (Torrens) title held by Tuason & Co. were truly void and
ineffective, then these facts should have been pleaded by these appellants in the previous case (Q4275), since such facts, if true, constituted a defense to the claim of Tuason & Co. for recovery of
possession. If appellants failed to plead such defenses in that previous case, they are barred from
litigating the same in any subsequent proceeding, for it is a well established rule that as between the
same parties and on the same subject and cause of action, a final judgment is conclusive not only on
matters directly adjudicated, but also as to any other matter that could have been raised in relation
thereto. 9
II
Did the respondent judge, in issuing the order, dated October 12, 1987, commit a grave abuse of
discretion equivalent to lack of excess of jurisdiction?
(a)
The Court has no doubt that Judge Velez is here guilty of grave abuse of discretion tantamount to lack
or excess of jurisdiction to warrant certiorari. As above-stated, what he gave away, by virtue of
reconveyance, was property that inalienably belongs to the Government or its successors. Worse, he
gave away property without notice to the actual possessors, that is, the present registered owner. It is
beyond debate, as we have indicated, that the land had been, since the cancellation of OCT No. 291,
parcelled out to a succession of buyers and owners. In the absence of notice, it acquired no jurisdiction
to decree redelivery or reconveyance. It is well-established that owners of property over which
reconveyance is asserted are indispensable parties, without whom no relief is available and without
whom the court can render no valid judgment. 10
Furthermore, the present holders of the land in question are innocent purchasers for value, or
presumed to be so in the absence of contrary evidence, against whom reconveyance does not lie.

11

(b)
The respondent judge can not conceal his faults behind arguments that he did not intend to convey the
premises, but rather, to secure, allegedly, vacant portions thereof from interlopers. First, this is not
stated in his order. Second, that order is clear and unequivocal that Domingo Palomares has the right
"[t]o sell, exchange, lease or otherwise dispose of any area or areas or portion or portions thereof . . .
" 12 Third and last, the security of the property is the lookout of the claimants, and not the court's. In
case the premises the respondent judge's injunctive writ have been directed belong to others, let them
air their plaints.
(c)

The Court is also agreed that the challenged order was issued with no benefit of trial or hearing. The
private respondent can not validly rely on AC-G.R. No. 00293 as the "trial or hearing" to justify the
issuance of its said order, in the first place, because it is a different proceeding. But above all, the
private respondent itself says that AC-G.R. CV No. 00293 can not be made a basis for denying
reconveyance because "the . . . petition was merely for the issuance of a new owner's duplicate
copy . . . 13 Accordingly, it can not invoke that case and yet, repudiate its effects. It is the height of
contradiction.
(d)
It was also grave error for the lower court to deny the Solicitor General's notice of appeal. The
Government had all the right to appeal because: (1) the order of October 12, 1987 was in the nature of
a final judgment, as "final judgment" is known in law (however it is captioned), that is to say, one that
"finally disposes of the pending action so that nothing more can be done with it in the trial court; 14 (2)
it did not merely maintain the status quo, but allowed Mr. Domingo Palomares to transact on the
property by near right of dominion over it.
Judge Velez had therefore no reason, indeed, excuse, to deny the Government's notice of appeal. What
is plain is the fact that Judge Velez was hell-bent, so to speak, in blocking the Government's efforts to
defend what rightfully belongs to it.
What has obviously been lost on the parties, Judge Velez in particular, is the established principle that
injunction does not lie "to take property out of the possession or control of one party and place it into
that of another." 15 In this wise it has also been held:
xxx

xxx

xxx

It is a well established doctrine in this jurisdiction that an injunction is not the proper remedy for the
recovery of possession of real estate and the improvements thereon, as well as for the ejectments
therefrom of the actual occupants who claim to have title to or material interest therein. The use of
said remedy in such cases has invariably been considered unjustified, in open violation of the legal
presumption that the bona fide possessor of a certain piece of land and improvements thereon, holds
the same under claim of ownership and with a just title, and as an advanced concession of the remedy
to which the claimant might be entitled. (Citations omitted) 16
xxx

xxx

xxx

Injunction, moreover, is an extraordinary remedy. It lies only in certain cases, to wit:


Sec. 3. Grounds for issuance of preliminary injunction. - A preliminary injunction may be granted at any
time after the commencement of the action and before judgment when it is established:
(a) That the plaintiff is entitled to the relief demanded, and the whole or part of such relief consists in
restraining the commission or continuance of the acts complained of, or in the performance of an act
or acts, either for a limited period or perpetually;
(b) That the commission or continuance of some act complained of during the litigation or the nonperformance thereof would probably work injustice to the plaintiff; or
(c) That the defendant is doing, threatens, or is about to do, or is procuring or suffering to be done,
some act probably in violation of the plaintiffs rights respecting the subject of the action, and tending
to render the judgment ineffectual. 17
xxx

xxx

xxx

The conspicuous and unusual zeal with which Judge Francisco Velez now defends his acts 18 has not
escaped us. His Honor should have borne in mind that in proceedings under Rule 65 of the Rules, such
as the present cases, the judge is included only as a nominal party. Unless otherwise ordained by this
Court, he is not called upon to answer or comment on the petition, but rather, the private respondent.
It is indeed distressing to note that it is the very judge who has taken the cudgels for the latter, in
defending its interests, when he, the judge, should have remained a neutral magistrate. Res ipsa
loquitor. 19 He must get his just deserts.
III
The Court thus closes the long-drawn tale of Hacienda de Maricaban. In this connection, let trial judges
be cautioned on the indiscriminate disposition of our dwindling natural resources to private persons.
Accordingly, we grant G.R. No. 81564 and dismiss G.R. No. 90176, and so also, end what has come
down as nearly a century of uncertainty, doubt, and conflict Maricaban has left in its trail. The Court
has finally spoken. Let the matter rest.
WHEREFORE:
1. The petition in G.R. No. 81564 is GRANTED:
(a) The Writ of Preliminary Injunction issued by our Resolution, dated April 13, 1988, enjoining the
respondent judge from enforcing his: (i) order of October 12, 1987 and (ii) the follow-up order of
October 23, 1987, is made permanent and
(b) Original Certificate of Title No. 291 is declared duly CANCELLED;
2. The petition in G.R. No. 90176 is DISMISSED; and
3. Judge Francisco Velez is ordered to SHOW CAUSE why he should not be administratively dealt with
for giving away, by virtue of reconveyance, property that inalienably belongs to the Government,
without notice to the registered owner, and without benefit of trial or hearing; for blocking Government
efforts to defend what rightfully belongs to it; and for filing his comment of June 17, 1988 and
supplemental comment of August 26, 1988 without express leave of court.
Costs against the private respondent.
SO ORDERED.
G.R. No. L-630

November 15, 1947

ALEXANDER A. KRIVENKO, petitioner-appellant,


vs.
THE REGISTER OF DEEDS, CITY OF MANILA, respondent and appellee.
Gibbs, Gibbs, Chuidian and Quasha of petitioner-appellant.
First Assistant Solicitor General Reyes and Solicitor Carreon for respondent-appellee.
Marcelino Lontok appeared as amicus curies.
MORAN, C.J.:
Alenxander A. Kriventor alien, bought a residential lot from the Magdalena Estate, Inc., in December of
1941, the registration of which was interrupted by the war. In May, 1945, he sought to accomplish said
registration but was denied by the register of deeds of Manila on the ground that, being an alien, he
cannot acquire land in this jurisdiction. Krivenko then brought the case to the fourth branch of the
Court of First Instance of Manila by means of a consulta, and that court rendered judgment sustaining
the refusal of the register of deeds, from which Krivenko appealed to this Court.

There is no dispute as to these facts. The real point in issue is whether or not an alien under our
Constitution may acquire residential land.
It is said that the decision of the case on the merits is unnecessary, there being a motion to withdraw
the appeal which should have been granted outright, and reference is made to the ruling laid down by
this Court in another case to the effect that a court should not pass upon a constitutional question if its
judgment may be made to rest upon other grounds. There is, we believe, a confusion of ideas in this
reasoning. It cannot be denied that the constitutional question is unavoidable if we choose to decide
this case upon the merits. Our judgment cannot to be made to rest upon other grounds if we have to
render any judgment at all. And we cannot avoid our judgment simply because we have to avoid a
constitutional question. We cannot, for instance, grant the motion withdrawing the appeal only
because we wish to evade the constitutional; issue. Whether the motion should be, or should not be,
granted, is a question involving different considerations now to be stated.
According to Rule 52, section 4, of the Rules of Court, it is discretionary upon this Court to grant a
withdrawal of appeal after the briefs have been presented. At the time the motion for withdrawal was
filed in this case, not only had the briefs been prensented, but the case had already been voted and
the majority decision was being prepared. The motion for withdrawal stated no reason whatsoever, and
the Solicitor General was agreeable to it. While the motion was pending in this Court, came the new
circular of the Department of Justice, instructing all register of deeds to accept for registration all
transfers of residential lots to aliens. The herein respondent-appellee was naturally one of the registers
of deeds to obey the new circular, as against his own stand in this case which had been maintained by
the trial court and firmly defended in this Court by the Solicitor General. If we grant the withdrawal, the
the result would be that petitioner-appellant Alexander A. Krivenko wins his case, not by a decision of
this Court, but by the decision or circular of the Department of Justice, issued while this case was
pending before this Court. Whether or not this is the reason why appellant seeks the withdrawal of his
appeal and why the Solicitor General readily agrees to that withdrawal, is now immaterial. What is
material and indeed very important, is whether or not we should allow interference with the regular
and complete exercise by this Court of its constitutional functions, and whether or not after having
held long deliberations and after having reached a clear and positive conviction as to what the
constitutional mandate is, we may still allow our conviction to be silenced, and the constitutional
mandate to be ignored or misconceived, with all the harmful consequences that might be brought
upon the national patromony. For it is but natural that the new circular be taken full advantage of by
many, with the circumstance that perhaps the constitutional question may never come up again before
this court, because both vendors and vendees will have no interest but to uphold the validity of their
transactions, and very unlikely will the register of deeds venture to disobey the orders of their superior.
Thus, the possibility for this court to voice its conviction in a future case may be remote, with the result
that our indifference of today might signify a permanent offense to the Constitution.
All thse circumstances were thoroughly considered and weighted by this Court for a number of days
and the legal result of the last vote was a denial of the motion withdrawing the appeal. We are thus
confronted, at this stage of the proceedings, with our duty, the constitutional question becomes
unavoidable. We shall then proceed to decide that question.
Article XIII, section 1, of the Constitutional is as follows:
Article XIII. Conservation and utilization of natural resources.
SECTION 1. All agricultural, timber, and mineral lands of the public domain, water, minerals, coal,
petroleum, and other mineral oils, all forces of potential energy, and other natural resources of the
Philippines belong to the State, and their disposition, exploitation, development, or utilization shall be
limited to citizens of the Philippines, or to corporations or associations at least sixty per centum of the
capital of which is owned by such citizens, subject to any existing right, grant, lease, or concession at
the time of the inaguration of the Government established uunder this Constitution. Natural resources,

with the exception of public agricultural land, shall not be alienated, and no licence, concession, or
lease for the exploitation, development, or utilization of any of the natural resources shall be granted
for a period exceeding twenty-five years, renewable for another twenty-five years, except as to water
rights for irrigation, water supply, fisheries, or industrial uses other than the development of water
"power" in which cases beneficial use may be the measure and the limit of the grant.
The scope of this constitutional provision, according to its heading and its language, embraces all
lands of any kind of the public domain, its purpose being to establish a permanent and fundamental
policy for the conservation and utilization of all natural resources of the Nation. When, therefore, this
provision, with reference to lands of the public domain, makes mention of only agricultural, timber and
mineral lands, it means that all lands of the public domain are classified into said three groups,
namely, agricultural, timber and mineral. And this classification finds corroboration in the circumstance
that at the time of the adoption of the Constitution, that was the basic classification existing in the
public laws and judicial decisions in the Philippines, and the term "public agricultural lands" under said
classification had then acquired a technical meaning that was well-known to the members of the
Constitutional Convention who were mostly members of the legal profession.
As early as 1908, in the case of Mapa vs. Insular Government (10 Phil., 175, 182), this Court said that
the phrase "agricultural public lands" as defined in the Act of Congress of July 1, 1902, which phrase is
also to be found in several sections of the Public Land Act (No. 926), means "those public lands
acquired from Spain which are neither mineral for timber lands." This definition has been followed in
long line of decisions of this Court. (SeeMontano vs. Insular Government, 12 Phil., 593; Ibaez de
Aldecoa vs. Insular Government, 13 Phil., 159; Ramosvs. Director of Lands, 39 Phil., 175;
Jocson vs. Director of Forestry, 39 Phil., 560; Ankron vs. Government of the Philippines, 40 Phil., 10.)
And with respect to residential lands, it has been held that since they are neither mineral nor timber
lands, of necessity they must be classified as agricultural. In Ibaez de Aldecoa vs. Insular Government
(13 Phil., 159, 163), this Court said:
Hence, any parcel of land or building lot is susceptible of cultivation, and may be converted into a field,
and planted with all kinds of vegetation; for this reason, where land is not mining or forestal in its
nature, it must necessarily be included within the classification of agricultural land, not because it is
actually used for the purposes of agriculture, but because it was originally agricultural and may again
become so under other circumstances; besides, the Act of Congress contains only three classification,
and makes no special provision with respect to building lots or urban lands that have ceased to be
agricultural land.
In other words, the Court ruled that in determining whether a parcel of land is agricultural, the test is
not only whether it is actually agricultural, but also its susceptibility to cultivation for agricultural
purposes. But whatever the test might be, the fact remains that at the time the Constitution was
adopted, lands of the public domain were classified in our laws and jurisprudence into agricultural,
mineral, and timber, and that the term "public agricultural lands" was construed as referring to those
lands that were not timber or mineral, and as including residential lands. It may safely be presumed,
therefore, that what the members of the Constitutional Convention had in mind when they drafted the
Constitution was this well-known classification and its technical meaning then prevailing.
Certain expressions which appear in Constitutions, . . . are obviously technical; and where such words
have been in use prior to the adoption of a Constitution, it is presumed that its framers and the people
who ratified it have used such expressions in accordance with their technical meaning. (11 Am. Jur.,
sec. 66, p. 683.) Also Calder vs. Bull, 3 Dall. [U.S.], 386; 1 Law. ed., 648; Bronson vs. Syverson, 88
Wash., 264; 152 P., 1039.)
It is a fundamental rule that, in construing constitutions, terms employed therein shall be given the
meaning which had been put upon them, and which they possessed, at the time of the framing and
adoption of the instrument. If a word has acquired a fixed, technical meaning in legal and

constitutional history, it will be presumed to have been employed in that sense in a written
Constitution. (McKinney vs. Barker, 180 Ky., 526; 203 S.W., 303; L.R.A., 1918 E, 581.)
Where words have been long used in a technical sense and have been judicially construed to have a
certain meaning, and have been adopted by the legislature as having a certain meaning prior to a
particular statute in which they are used, the rule of construction requires that the words used in such
statute should be construed according to the sense in which they have been so previously used,
although the sense may vary from strict literal meaning of the words. (II Sutherland, Statutory
Construction, p. 758.)
Therefore, the phrase "public agricultural lands" appearing in section 1 of Article XIII of the Constitution
must be construed as including residential lands, and this is in conformity with a legislative
interpretation given after the adoption of the Constitution. Well known is the rule that "where the
Legislature has revised a statute after a Constitution has been adopted, such a revision is to be
regarded as a legislative construction that the statute so revised conforms to the Constitution." (59
C.J., 1102.) Soon after the Constitution was adopted, the National Assembly revised the Public Land
Law and passed Commonwealth Act No. 141, and sections 58, 59 and 60 thereof permit the sale of
residential lots to Filipino citizens or to associations or corporations controlled by such citizens, which is
equivalent to a solemn declaration that residential lots are considered as agricultural lands, for, under
the Constitution, only agricultural lands may be alienated.
It is true that in section 9 of said Commonwealth Act No. 141, "alienable or disposable public lands"
which are the same "public agriculture lands" under the Constitution, are classified into agricultural,
residential, commercial, industrial and for other puposes. This simply means that the term "public
agricultural lands" has both a broad and a particular meaning. Under its broad or general meaning, as
used in the Constitution, it embraces all lands that are neither timber nor mineral. This broad meaning
is particularized in section 9 of Commonwealth Act No. 141 which classifies "public agricultural lands"
for purposes of alienation or disposition, into lands that are stricly agricultural or actually devoted to
cultivation for agricultural puposes; lands that are residential; commercial; industrial; or lands for other
purposes. The fact that these lands are made alienable or disposable under Commonwealth Act No.
141, in favor of Filipino citizens, is a conclusive indication of their character as public agricultural lands
under said statute and under the Constitution.
It must be observed, in this connection that prior to the Constitution, under section 24 of Public Land
Act No. 2874, aliens could acquire public agricultural lands used for industrial or residential puposes,
but after the Constitution and under section 23 of Commonwealth Act No. 141, the right of aliens to
acquire such kind of lands is completely stricken out, undoubtedly in pursuance of the constitutional
limitation. And, again, prior to the Constitution, under section 57 of Public Land Act No. 2874, land of
the public domain suitable for residence or industrial purposes could be sold or leased to aliens, but
after the Constitution and under section 60 of Commonwealth Act No. 141, such land may only be
leased, but not sold, to aliens, and the lease granted shall only be valid while the land is used for the
purposes referred to. The exclusion of sale in the new Act is undoubtedly in pursuance of the
constitutional limitation, and this again is another legislative construction that the term "public
agricultural land" includes land for residence purposes.
Such legislative interpretation is also in harmony with the interpretation given by the Executive
Department of the Government. Way back in 1939, Secretary of Justice Jose Abad Santos, in answer to
a query as to "whether or not the phrase 'public agricultural lands' in section 1 of Article XII (now XIII)
of the Constitution may be interpreted to include residential, commercial, and industrial lands for
purposes of their disposition," rendered the following short, sharp and crystal-clear opinion:
Section 1, Article XII (now XIII) of the Constitution classifies lands of the public domain in the
Philippines into agricultural, timber and mineral. This is the basic classification adopted since the
enactment of the Act of Congress of July 1, 1902, known as the Philippine Bill. At the time of the

adoption of the Constitution of the Philippines, the term 'agricultural public lands' and, therefore,
acquired a technical meaning in our public laws. The Supreme Court of the Philippines in the leading
case of Mapa vs. Insular Government, 10 Phil., 175, held that the phrase 'agricultural public lands'
means those public lands acquired from Spain which are neither timber nor mineral lands. This
definition has been followed by our Supreme Court in many subsequent case. . . .
Residential commercial, or industrial lots forming part of the public domain must have to be included in
one or more of these classes. Clearly, they are neither timber nor mineral, of necessity, therefore, they
must be classified as agricultural.
Viewed from another angle, it has been held that in determining whether lands are agricultural or not,
the character of the land is the test (Odell vs. Durant, 62 N.W., 524; Lorch vs. Missoula Brick and Tile
Co., 123 p.25). In other words, it is the susceptibility of the land to cultivation for agricultural purposes
by ordinary farming methods which determines whether it is agricultural or not (State vs. Stewart, 190
p. 129).
Furthermore, as said by the Director of Lands, no reason is seen why a piece of land, which may be
sold to a person if he is to devote it to agricultural, cannot be sold to him if he intends to use it as a
site for his home.
This opinion is important not alone because it comes from a Secratary of Justice who later became the
Chief Justice of this Court, but also because it was rendered by a member of the cabinet of the late
President Quezon who actively participated in the drafting of the constitutional provision under
consideration. (2 Aruego, Framing of the Philippine Constitution, p. 598.) And the opinion of the Quezon
administration was reiterated by the Secretary of Justice under the Osmea administration, and it was
firmly maintained in this Court by the Solicitor General of both administrations.
It is thus clear that the three great departments of the Government judicial, legislative and
executive have always maintained that lands of the public domain are classified into agricultural,
mineral and timber, and that agricultural lands include residential lots.
Under section 1 of Article XIII of the Constitution, "natural resources, with the exception of public
agricultural land, shall not be aliented," and with respect to public agricultural lands, their alienation is
limited to Filipino citizens. But this constitutional purpose conserving agricultural resources in the
hands of Filipino citizens may easily be defeated by the Filipino citizens themselves who may alienate
their agricultural lands in favor of aliens. It is partly to prevent this result that section 5 is included in
Article XIII, and it reads as follows:
Sec. 5. Save in cases of hereditary succession, no private agricultural land will be transferred or
assigned except to individuals, corporations, or associations qualified to acquire or hold lands of the
public domain in the Philippines.
This constitutional provision closes the only remaining avenue through which agricultural resources
may leak into aliens' hands. It would certainly be futile to prohibit the alienation of public agricultural
lands to aliens if, after all, they may be freely so alienated upon their becoming private agricultural
lands in the hands of Filipino citizens. Undoubtedly, as above indicated, section 5 is intended to insure
the policy of nationalization contained in section 1. Both sections must, therefore, be read together for
they have the same purpose and the same subject matter. It must be noticed that the persons against
whom the prohibition is directed in section 5 are the very same persons who under section 1 are
disqualified "to acquire or hold lands of the public domain in the Philippines." And the subject matter of
both sections is the same, namely, the non-transferability of "agricultural land" to aliens. Since
"agricultural land" under section 1 includes residential lots, the same technical meaning should be
attached to "agricultural land under section 5. It is a rule of statutory construction that "a word or
phrase repeated in a statute will bear the same meaning throughout the statute, unless a different
intention appears." (II Sutherland, Statutory Construction, p. 758.) The only difference between

"agricultural land" under section 5, is that the former is public and the latter private. But such
difference refers to ownership and not to the class of land. The lands are the same in both sections,
and, for the conservation of the national patrimony, what is important is the nature or class of the
property regardless of whether it is owned by the State or by its citizens.
Reference is made to an opinion rendered on September 19, 1941, by the Hon. Teofilo Sison, then
Secretary of Justice, to the effect that residential lands of the public domain may be considered as
agricultural lands, whereas residential lands of private ownership cannot be so considered. No reason
whatsoever is given in the opinion for such a distinction, and no valid reason can be adduced for such
a discriminatory view, particularly having in mind that the purpose of the constitutional provision is the
conservation of the national patrimony, and private residential lands are as much an integral part of
the national patrimony as the residential lands of the public domain. Specially is this so where, as
indicated above, the prohibition as to the alienable of public residential lots would become superflous if
the same prohibition is not equally applied to private residential lots. Indeed, the prohibition as to
private residential lands will eventually become more important, for time will come when, in view of
the constant disposition of public lands in favor of private individuals, almost all, if not all, the
residential lands of the public domain shall have become private residential lands.
It is maintained that in the first draft of section 5, the words "no land of private ownership" were used
and later changed into "no agricultural land of private ownership," and lastly into "no private
agricultural land" and from these changes it is argued that the word "agricultural" introduced in the
second and final drafts was intended to limit the meaning of the word "land" to land actually used for
agricultural purposes. The implication is not accurate. The wording of the first draft was amended for
no other purpose than to clarify concepts and avoid uncertainties. The words "no land" of the first
draft, unqualified by the word "agricultural," may be mistaken to include timber and mineral lands, and
since under section 1, this kind of lands can never be private, the prohibition to transfer the same
would be superfluous. Upon the other hand, section 5 had to be drafted in harmony with section 1 to
which it is supplementary, as above indicated. Inasmuch as under section 1, timber and mineral lands
can never be private, and the only lands that may become private are agricultural lands, the words "no
land of private ownership" of the first draft can have no other meaning than "private agricultural land."
And thus the change in the final draft is merely one of words in order to make its subject matter more
specific with a view to avoiding the possible confusion of ideas that could have arisen from the first
draft.
If the term "private agricultural lands" is to be construed as not including residential lots or lands not
strictly agricultural, the result would be that "aliens may freely acquire and possess not only residential
lots and houses for themselves but entire subdivisions, and whole towns and cities," and that "they
may validly buy and hold in their names lands of any area for building homes, factories, industrial
plants, fisheries, hatcheries, schools, health and vacation resorts, markets, golf courses, playgrounds,
airfields, and a host of other uses and purposes that are not, in appellant's words, strictly agricultural."
(Solicitor General's Brief, p. 6.) That this is obnoxious to the conservative spirit of the Constitution is
beyond question.
One of the fundamental principles underlying the provision of Article XIII of the Constitution and which
was embodied in the report of the Committee on Nationalization and Preservation of Lands and other
Natural Resources of the Constitutional Convention, is "that lands, minerals, forests, and other natural
resources constitute the exclusive heritage of the Filipino nation. They should, therefore, be preserved
for those under the sovereign authority of that nation and for their posterity." (2 Aruego, Framing of the
Filipino Constitution, p. 595.) Delegate Ledesma, Chairman of the Committee on Agricultural
Development of the Constitutional Convention, in a speech delivered in connection with the national
policy on agricultural lands, said: "The exclusion of aliens from the privilege of acquiring public
agricultural lands and of owning real estate is a necessary part of the Public Land Laws of the
Philippines to keep pace with the idea of preserving the Philippines for the Filipinos." (Emphasis ours.)
And, of the same tenor was the speech of Delegate Montilla who said: "With the complete

nationalization of our lands and natural resources it is to be understood that our God-given birthright
should be one hundred per cent in Filipino hands . . .. Lands and natural resources are immovables and
as such can be compared to the vital organs of a person's body, the lack of possession of which may
cause instant death or the shortening of life. If we do not completely antionalize these two of our most
important belongings, I am afraid that the time will come when we shall be sorry for the time we were
born. Our independence will be just a mockery, for what kind of independence are we going to have if
a part of our country is not in our hands but in those of foreigners?" (Emphasis ours.) Professor Aruego
says that since the opening days of the Constitutional Convention one of its fixed and dominating
objectives was the conservation and nationalization of the natural resources of the country. (2 Aruego,
Framing of the Philippine Constitution, p 592.) This is ratified by the members of the Constitutional
Convention who are now members of this Court, namely, Mr. Justice Perfecto, Mr. Justice Briones, and
Mr. Justice Hontiveros. And, indeed, if under Article XIV, section 8, of the Constitution, an alien may not
even operate a small jitney for hire, it is certainly not hard to understand that neither is he allowed to
own a pieace of land.
This constitutional intent is made more patent and is strongly implemented by an act of the National
Assembly passed soon after the Constitution was approved. We are referring again to Commonwealth
Act No. 141. Prior to the Constitution, there were in the Public Land Act No. 2874 sections 120 and 121
which granted aliens the right to acquire private only by way of reciprocity. Said section reads as
follows:
SEC. 120. No land originally acquired in any manner under the provisions of this Act, nor any
permanent improvement on such land, shall be encumbered, alienated, or transferred, except to
persons, corporations, associations, or partnerships who may acquire lands of the public domain under
this Act; to corporations organized in the Philippine Islands authorized therefor by their charters, and,
upon express authorization by the Philippine Legislature, to citizens of countries the laws of which
grant to citizens of the Philippine Islands the same right to acquire, hold, lease, encumber, dispose of,
or alienate land, or permanent improvements thereon, or any interest therein, as to their own citizens,
only in the manner and to the extent specified in such laws, and while the same are in force but not
thereafter.
SEC. 121. No land originally acquired in any manner under the provisions of the former Public Land Act
or of any other Act, ordinance, royal order, royal decree, or any other provision of law formerly in force
in the Philippine Islands with regard to public lands, terrenos baldios y realengos, or lands of any other
denomination that were actually or presumptively of the public domain or by royal grant or in any
other form, nor any permanent improvement on such land, shall be encumbered, alienated, or
conveyed, except to persons, corporations, or associations who may acquire land of the public domain
under this Act; to corporate bodies organized in the Philippine Islands whose charters may authorize
them to do so, and, upon express authorization by the Philippine Legislature, to citizens of the
countries the laws of which grant to citizens of the Philippine Islands the same right to acquire, hold,
lease, encumber, dispose of, or alienate land or pemanent improvements thereon or any interest
therein, as to their own citizens, and only in the manner and to the extent specified in such laws, and
while the same are in force, but not thereafter:Provided, however, That this prohibition shall not be
applicable to the conveyance or acquisition by reason of hereditary succession duly acknowledged and
legalized by competent courts, nor to lands and improvements acquired or held for industrial or
residence purposes, while used for such purposes:Provided, further, That in the event of the ownership
of the lands and improvements mentioned in this section and in the last preceding section being
transferred by judicial decree to persons,corporations or associations not legally capacitated to acquire
the same under the provisions of this Act, such persons, corporations, or associations shall be obliged
to alienate said lands or improvements to others so capacitated within the precise period of five years,
under the penalty of such property reverting to the Government in the contrary case." (Public Land Act,
No. 2874.)

It is to be observed that the pharase "no land" used in these section refers to all private lands, whether
strictly agricultural, residential or otherwise, there being practically no private land which had not been
acquired by any of the means provided in said two sections. Therefore, the prohibition contained in
these two provisions was, in effect, that no private land could be transferred to aliens except "upon
express authorization by the Philippine Legislature, to citizens of Philippine Islands the same right to
acquire, hold, lease, encumber, dispose of, or alienate land." In other words, aliens were granted the
right to acquire private land merely by way of reciprocity. Then came the Constitution and
Commonwealth Act No. 141 was passed, sections 122 and 123 of which read as follows:
SEC. 122. No land originally acquired in any manner under the provisions of this Act, nor any
permanent improvement on such land, shall be encumbered, alienated, or transferred, except to
persons, corporations, associations, or partnerships who may acquire lands of the public domain under
this Act or to corporations organized in the Philippines authorized thereof by their charters.
SEC. 123. No land originally acquired in any manner under the provisions of any previous Act,
ordinance, royal order, royal decree, or any other provision of law formerly in force in the Philippines
with regard to public lands terrenos baldios y realengos, or lands of any other denomination that were
actually or presumptively of the public domain, or by royal grant or in any other form, nor any
permanent improvement on such land, shall be encumbered, alienated, or conveyed, except to
persons, corporations or associations who may acquire land of the public domain under this Act or to
corporate bodies organized in the Philippines whose charters authorize them to do so: Provided,
however, That this prohibition shall not be applicable to the conveyance or acquisition by reason of
hereditary succession duly acknowledged and legalized by competent courts: Provided, further, That in
the event of the ownership of the lands and improvements mentioned in this section and in the last
preceding section being transferred by judicial decree to persons, corporations or associations not
legally capacitated to acquire the same under the provisions of this Act, such persons, corporations, or
associations shall be obliged to alienate said lands or improvements to others so capacitated within
the precise period of five years; otherwise, such property shall revert to the Government.
These two sections are almost literally the same as sections 120 and 121 of Act No. 2874, the only
difference being that in the new provisions, the right to reciprocity granted to aliens is completely
stricken out. This, undoubtedly, is to conform to the absolute policy contained in section 5 of Article XIII
of the Constitution which, in prohibiting the alienation of private agricultural lands to aliens, grants
them no right of reciprocity. This legislative construction carries exceptional weight, for prominent
members of the National Assembly who approved the new Act had been members of the Constitutional
Convention.
It is said that the lot question does not come within the purview of sections 122 and 123 of
Commonwealth Act No. 141, there being no proof that the same had been acquired by one of the
means provided in said provisions. We are not, however, diciding the instant case under the provisions
of the Public Land Act, which have to refer to land that had been formerly of the public domain,
otherwise their constitutionality may be doubtful. We are deciding the instant case under section 5 of
Article XIII of the Constitution which is more comprehensive and more absolute in the sense that it
prohibits the transfer to alien of any private agricultural land including residential land whatever its
origin might have been.
And, finally, on June 14, 1947, the Congress approved Republic Act No. 133 which allows mortgage of
"private real property" of any kind in favor of aliens but with a qualification consisting of expressly
prohibiting aliens to bid or take part in any sale of such real property as a consequence of the
mortgage. This prohibition makes no distinction between private lands that are strictly agricultural and
private lands that are residental or commercial. The prohibition embraces the sale of private lands of
any kind in favor of aliens, which is again a clear implementation and a legislative interpretation of the
constitutional prohibition. Had the Congress been of opinion that private residential lands may be sold
to aliens under the Constitution, no legislative measure would have been found necessary to authorize

mortgage which would have been deemed also permissible under the Constitution. But clearly it was
the opinion of the Congress that such sale is forbidden by the Constitution and it was such opinion that
prompted the legislative measure intended to clarify that mortgage is not within the constitutional
prohibition.
It is well to note at this juncture that in the present case we have no choice. We are construing the
Constitution as it is and not as we may desire it to be. Perhaps the effect of our construction is to
preclude aliens, admitted freely into the Philippines from owning sites where they may build their
homes. But if this is the solemn mandate of the Constitution, we will not attempt to compromise it
even in the name of amity or equity. We are satisfied, however, that aliens are not completely excluded
by the Constitution from the use of lands for residential purposes. Since their residence in the
Philippines is temporary, they may be granted temporary rights such as a lease contract which is not
forbidden by the Constitution. Should they desire to remain here forever and share our fortunes and
misfortunes, Filipino citizenship is not impossible to acquire.
For all the foregoing, we hold that under the Constitution aliens may not acquire private or public
agricultural lands, including residential lands, and, accordingly, judgment is affirmed, without costs.
Feria, Pablo, Perfecto, Hilado, and Briones, JJ., concur.
Collado vs CA Oct. 4, 2002
G.R. No. L-33676 June 30, 1971
MARIANO PAJOMAYO, PATRICIO PAJOMAYO, EUSEBIO PAJOMAYO, SOLEDAD PAJOMAYO
assisted by her husband FLORIANO CHITONGCO, DEMFTRIO PAJOMAYO, CRISTITA PAJOMAYO
assisted by her husband MANUEL RAMIREZ, PATROCINIO PAJOMAYO and CRISPO
PAJOMAYO, plaintiffs-appellees,
vs.
RODRIGO MANIPON and PERFECTA ZULUETA, defendants-appellants.
.
ZALDIVAR, J.:
Appeal from the decision of the Court of First Instance of Pangasinan (Branch IX) in its Civil Case No. U655. The decision was originally appealed to the Court of Appeals on November 3, 1964. In the
resolution of the second special division of the Court of Appeals, promulgate on April 27, 1971, this
case was certified to this Court as one that is within the exclusive appellate jurisdiction of the Supreme
Court only errors on question of law being involved in the appeal. 1
On June 5, 1963 the plaintiffs filed in the Court of First Instance of Pangasinan (Branch IX, at Urdaneta,
Pangasinan) a complaint alleging that they are owners pro-indiviso of the parcel of land described in
the complaint which is covered by Original Certificate of Title No. 1089 in the name of Diego Pajomayo,
issued by the office of the Register of Deeds of Pangasinan; that they had acquired the land as an
inheritance from their late father Diego Pajomayo; that they and their predecessor-in-interest had been
in actual, peaceful and uninterrupted possession of said property in the concept of owners for a period
of more than 70 years until the early part of the year 1956 when the defendants dispossessed them of
said property, resulting in their having suffered annual damages amounting to around P1,100.00
representing the value of the crops of rice; mongo, corn and vegetables that they failed to harvest; and
that because they have to file the present suit they must spend P800.00 for incidental expenses of
litigation and attorney's fees. The plaintiffs prayed that they be declared the lawful owners pro-indiviso
of the land in question, and that the defendants be ordered to vacate the land and pay them the
damages they have suffered.

In their answer the defendants, after denying some of the allegations of the complaint, alleged that
they are the exclusive owners of a parcel of land covered by Original Certificate of Title No. 14043
issued by the office of the Register of Deeds of Pangasinan, the said land having been adjudicated to
them in the cadastral proceedings of the Malasique cadastre and that apparently the plaintiffs are
claiming the same parcel of land. The defendants claim they had acquired the land mentioned in their
answer by inheritance from their deceased father Pioquinto Manipon, and that they and their
predecessors-in-interest have been in actual, peaceful, and adverse possession of said land for more
than 70 years, to the reclusion of plaintiffs; and that as possessors in good faith they have introduced
on the land improvements worth P1,000.00. As affirmative defenses, the defendants allege that
plaintiffs' action is barred by res-judicata and/or prescription and that the court has no jurisdiction over
the subject matter of the case. The defendants set up a counterclaim for damages in the sum of
P500.00 representing attorney's fees that they paid their counsel. The defendants prayed that they be
declared the owners of the parcel of land mentioned in their answer; that the plaintiffs be ordered to
pay them damages in the sum of P500.00; and, in the alternative should judgment be rendered against
them, that the plaintiffs ordered jointly and severally to pay them the sum of P1,000.00 representing
the value of the improvements they have introduced on the land.
When the case, was called for trial on July 6, 1964, the counsels for the parties submitted to the court
a stipulation of facts, as follows:
1. That plaintiffs are the children and compulsory heirs of the late Diego Pajomayo;
2. That parties agree that the land in question is covered by two Certificates of Title, one in the name
of Diego Pajomayo under Original Cert. of Title No. 1089 issued under Free Patent, owner's copy
attached hereto as Annex A; and Original Cert. of Title No. 14034, in the name of the Defendant
Rodrigo Manipon, issued in Cadastral Case No. 91 of Malasique Cadastre, certified true copy of which is
attached hereto as Annex B;
3. That parties agree to submit this case on the above stipulations without further presentation of
evidence.
WHEREFORE, it is respectfully prayed this Honorable Court that decision be rendered upon the
foregoing stipulation after the parties have submitted simultaneous memoranda within a period of
twenty (20) days from today.
Urdaneta, Pangasinan this 6th day of July, 1964.
On the basis of the foregoing stipulation of facts, the Court of First Instance of Pangasinan (Branch IX)
made a finding that Original Certificate of Title No. 1089 held by the plaintiffs was issued earlier than
Original Certificate of Title No. 14034 held by the defendants, and on September 10, 1964 it rendered
a decision, the dispositive portion of which reads as follows:
WHEREFORE, the Court, rendering judgment in favor of the plaintiffs and against the defendants,
hereby orders the latter to vacate the land in question and deliver possession thereof to the former
who are entitled thereto as the heirs of Diego Pajomayo who is hereby declared the legal and lawful
owner of the said property.
The Register of Deeds for Pangasinan is hereby ordered to cancel de oficio Original Certificate of Title
No. 14034.
With costs of this suit against the defendant.
From the above-mentioned decision of the lower court, the defendants brought up the present appeal.
In their appeal the defendants made the following assignment of errors:

1. The lower court erred in declaring Original Certificate of Title No. 14034 of herein appellants null and
void notwithstanding the fact that this is not one of the reliefs prayed for by the appellees.
2. The lower erred in ordering the herein appellants to vacate the land in question and to deliver the
possession thereof to the herein appellees although the latter failed to prove their cause of action
against the herein appellants.
3. The lower court erred in not applying the doctrine of res judicata in favor of herein appellants.
The appeal has no merit. There is no question regarding the identity of the land involved. The only
question to be resolved in the present appeal is: which of the two original certificates of title should
prevail the No. 1089 held by the plaintiffs-appellees which was issued in virtue of the homestead
patent, or the No. 14034 held by the defendants-appellants which was issued in connection with the
cadastral proceedings? Necessarily when one of the two titles is held to be superior over the other, one
should be declared null and void and should be ordered cancelled. And if a party is declared to be the
owner of a parcel of land pursuant to a valid certificate of title said party is entitled to the possession
of the land covered by said valid title. The decree of registration issued in the cadastral proceedings
does not have the effect of annulling the title that had previously been issued in accordance with the
provisions of the land Registration Law (Act 496).
The lower court, therefore, had correctly ordered the cancellation of Certificate of Title No 14034 held
by the defendants when it declared that Original Certificate of Title No. 1089 held by the plaintiffs
should prevail. Likewise, the lower court had correctly ordered the defendants to vacate the land in
question and deliver possession thereof to plaintiffs after declaring plaintiffs entitled thereto as the
heirs of Diego Pajomayo, the lawful owner of the land.
Contrary to the claim of defendants, the doctrine of res judicata can not be applied in their favor in the
present case.
The undisputed fact is that the plaintiffs base their claim of title to the land in question on Original
Certificate of Title No. 1089 issued to their father, Diego Pajomayo, on November 27, 1931 in virtue of
a free patent that was granted to him. The law requires that the homestead patent must be registered
in the office of the Register of Deeds of the province where the land covered by the patent lies. Section
122 of the Land Registration Act (Act 496) provides as follows:
SEC. 122. Whenever public lands in the Philippine Islands belonging (to the Government of the United
States or) to the Government of the Philippine Islands are alienated, granted, or conveyed to persons
or to public or private corporations, the same shall be brought forthwith under the operation of this Act
and shall become registered lands. It shall be the duty of the official issuing the instrument of
alienation, grant, or conveyance in behalf of the Government to came suck instrument before its
delivery to the grantee, to be filed with the register of deeds for the province where the land lies and
to be there registered like other deeds and conveyance, whereupon a certificate shall be entered as in
other cases of registered land and an owner's duplicate certificate issued to the grantee. The deed,
grant, or instrument of conveyance from the Government to the grantee shall not take effect as a
conveyance or bind the land, but shall operate only as contract between the Government and the
grantee and as evidence of authority to the clerk or register of deeds to make registration. The act of
registration shall be the operative act to convey and effect the land, and in all cases under this Act
registration shall be made in the office of the register of deeds for the province where the land lies.
The fees for registration shall be paid by the grantee. After due registration and issue of the certificate
and owner's duplicate, such land shall be registered land for all purposes under this Act. (Emphasis
supplied).
Thus, it has been ruled by this Court that once a homestead patent granted in accordance with the
Public Land Act registered pursuant to Section 122 of Act 496 (Land Registration Act), the certificate of

title issued in virtue of said patent has the force and effect of a Torrens Title under the Land
Registration Act. In the cage of Aquino vs. Director of Lands, 39 Phil. 850, this Court held:
The procedure under the Land Registration Law and under the provisions of Chapter VI of the Public
Land Law are the same in that both are against the whole world, both take the nature of judicial
proceedings, and for both the decree of registration issued is conclusive and final. (Act No. 496, secs.
35, 38, and 45, as amended; Act 926. secs. 59 and 63, as amended; Escueta vs. Director of Lands, 16
Phil. 482; Grey Alba vs. De la Cruz, 17 Phil. 49; Roxas vs. Enriquez, 29 Phil. 31; Legarda, et al. vs.
Saleeby, 31 Phil. 591) ... 2
In the case of Manalo vs. Lukban and Liwanag, 48 Phil. 973, 979, this Court said:
The record shows that the land covered by said judgment had already been granted by the
government to Monico Corpus Manuel as homesteader under the provisions of Act 926, the
corresponding certificate of title having been registered and issued to said grantee. By virtue of said
registration and issuance of the certificate of title, that land is considered registered within the
meaning of the Land Registration Act, No. 496 (sec. 122 of said Act).
So that when the trial was held in the cadastral proceeding which covered said land, and when the
judgment of June 29, 1922 concerning said land was rendered in said proceeding, the title to that land
could no longer be the subject of any inquiry, determination or judgment, for it had already been
adjudicated to Monico Corpus Manuel more than ten years before, with all the legal formalities and
with all the force of a title under Act 496.
The doctrine laid down in the two cases above-cited has been affirmed and applied by this Court in a
long line of decisions. 3 The ruling regarding the validity and force of a certificate of title issued in virtue
of the registration of a homestead patent is applicable to certificates of title issued in virtue of the
registration of other land patents under the Public land Law. In the case of Lahora, et al. vs.
Dayanghirang, et al., G.R. No. L-28565, January 30, 1971, 4 thus Court, speaking through Mr. Justice
J.B.L. Reyes, held:
The rule in this jurisdiction, regarding public land patents and the character of the certificate of title
that may be issued by virtue thereof, is that where land is granted by the government to a private
individual, the corresponding patent therefor is recorded, and the certificate of title is issued to the
grantee; thereafter, the land is automatically brought within the operation of the Land Registration Act,
the title issued to the grantee becoming entitled to all the safeguards provided in Section 38 of said
Act. In other words, upon the expiration of one year from its issuance, the certificate of title becomes
irrevocable and indefeasible like a certificate issued in a registration proceeding.
It is the settled rule in this jurisdiction that where two certificates of title are issued to different persons
covering the same land in whole or in part, the earlier in date must prevail as between the original
parties, and in case of successive registration where, more than one certificate is issued over the land
the person holding under the prior certificate is entitled to the land as against the person who relies on
the second certificate. 5
In the case now before Us, it appearing that Original Certificate of Title No. 14034 upon which the
defendant appellants base their claim of ownership over the land in question was issued on April 1,
1957, while Original Certificate of Title No. 1089 upon which plaintiffs-appellees base a similar claim
was issued on November 27, 1931, under the law and the authorities. We have herein cited, the latter
certificate of title should prevail, and the former should be cancelled.
WHEREFORE, the decision appealed from should be, as it is hereby, affirmed, with costs against the
defendants-appellants. It is so ordered.

Concepcion, C.J., Reyes, J.B.L Dizon, Makalintal, Fernando, Teehankee, Barredo, Villamor and Makasiar,
JJ., concur.
Castro, J., is on leave.

Das könnte Ihnen auch gefallen